Podcast Archives | The Audio PANCE and PANRE https://podcast.thepalife.com/category/podcast/ Physician Assistant Board Review Podcast Tue, 15 Aug 2023 04:14:57 +0000 en-US hourly 1 http://podcast.thepalife.com/wp-content/uploads/2017/04/cropped-The-Audio-PANCE-and-PANRE-512x512-32x32.jpg Podcast Archives | The Audio PANCE and PANRE https://podcast.thepalife.com/category/podcast/ 32 32 The Audio PANCE and PANRE is an audio Board and Rotation Review Series that includes ten Multiple Choice PANCE and PANRE Board Review Questions in Each Episode. Now you can study for your PANCE, PANRE, and End or Rotation Exams in the gym, in the car, on a run, or while relaxing on the beach. This free series is limited to every other episode. To download the complete series join Smarty PANCE at https://smartypance.com/sign-up/ The Physician Assistant Life | Smarty PANCE clean The Physician Assistant Life | Smarty PANCE The PA Life, INC The PA Life, INC podcast Multiple Choice Physician Assistant Board and Rotation Review Questions on the Go - Listen and Learn Podcast Archives | The Audio PANCE and PANRE http://podcast.thepalife.com/wp-content/uploads/powerpress/THE_AUDIO_PANCE_AND_PANRE_ACADEMY_PODCAST.png http://podcast.thepalife.com/category/podcast/ TV-G c9c7bad3-4712-514e-9ebd-d1e208fa1b76 78572745 Podcast Episode 107: This vs. That – PANCE Blueprint Comparisons You Need to Know (Part 1) http://podcast.thepalife.com/episode-107/ Tue, 15 Aug 2023 10:00:36 +0000 https://podcast.thepalife.com/?p=591 Listen to Podcast Episode 107: This vs. That – PANCE Blueprint Comparisons You Need to Know (Episode 1) In today’s session, we will be discussing five questions related to PANCE/PANRE Blueprint topics. These questions will cover similar presentations and crucial comparisons that are important for you to know. These topics are often used by PANCE/PANRE […]

The post Podcast Episode 107: This vs. That – PANCE Blueprint Comparisons You Need to Know (Part 1) appeared first on The Audio PANCE and PANRE.

]]>
Listen to Podcast Episode 107: This vs. That – PANCE Blueprint Comparisons You Need to Know (Episode 1)

In today’s session, we will be discussing five questions related to PANCE/PANRE Blueprint topics. These questions will cover similar presentations and crucial comparisons that are important for you to know. These topics are often used by PANCE/PANRE test question writers, so it’s essential to learn how to differentiate between them. This is the first part (episode 1) of a series.

If you can’t see the audio player, click here to listen to the full episode.

Podcast Episode 107 This vs. That - PANCE Blueprint Comparisons You Need to Know (Part 1)

Links from today’s episode:

I hope you enjoy this free audio component of the examination portion of this site. Smarty PANCE includes over 2,000 interactive board review questions, along with flashcards, ReelDx cases, integrated Picmonics, and lessons covering every blueprint topic available to all Smarty PANCE members.

Interactive exam to complement today’s podcast

1. A 32-year-old woman presents with a 6-month history of loose bowel movements, approximately eight per day. Blood has been present in many of them. She has lost 30 pounds. For the past 6 weeks, she has had intermittent fever. She has had no previous gastrointestinal (GI) problems, and there is no family history of GI problems. On examination, the patient looks ill. Her blood pressure is 130/ 70 mm Hg. Her pulse is 108 beats/ minute and regular. There is generalized abdominal tenderness with no rebound. A sigmoidoscopy reveals a friable rectal mucosa with multiple bleeding points. Which of the following is the most likely diagnosis?

A) Crohn’s Disease
B) Ulcerative Colitis
C) Infectious Colitis
D) Irritable Bowel Syndrome (IBS)
E) Ischemic Colitis

Answer and topic summary

The answer is B) Ulcerative Colitis

The patient’s symptoms of chronic bloody diarrhea, weight loss, fever, and the sigmoidoscopy findings of a friable rectal mucosa with multiple bleeding points are consistent with a diagnosis of ulcerative colitis (UC), which is a form of inflammatory bowel disease (IBD). UC typically involves the rectum and may extend proximally to involve other parts of the colon.

Incorrect answers:

A) Crohn’s Disease: This is another type of IBD. However, Crohn’s usually presents with non-bloody diarrhea, abdominal pain, and may involve any part of the GI tract from mouth to anus, often with skip lesions. In this case, the bloody diarrhea and the findings on sigmoidoscopy are more indicative of ulcerative colitis.

C) Infectious Colitis: Although infectious causes can lead to similar symptoms, the duration of this patient’s symptoms (6 months) is much longer than typically seen with infectious colitis. Additionally, fever is less common in infectious colitis.

D) Irritable Bowel Syndrome (IBS): IBS is a functional GI disorder characterized by abdominal pain with a change in bowel habit. It does not cause weight loss, fever, or bloody stools.

E) Ischemic Colitis: This typically presents acutely in older patients or those with vascular risk factors. The clinical presentation often includes abrupt onset of abdominal pain and bloody diarrhea. The duration and pattern of symptoms in this patient are more consistent with IBD.

Smarty PANCE Content Blueprint Review:

Covered under ⇒ PANCE Blueprint GI and NutritionColorectal disordersInflammatory bowel disease


2. A 27-year-old female presents to the emergency department with a 3-day history of a widespread painful rash. She reports having started a new medication for her seizures 1 week ago. On examination, you note erythematous macules that are coalescing into large areas of epidermal detachment. The mucous membranes of her mouth, eyes, and genital region are also affected, and the skin involvement covers more than 30% of her body surface area. A skin biopsy reveals full-thickness epidermal necrosis. Based on her presentation and the medication history, which of the following diagnoses is she most likely suffering from, and which medication most likely contributed to this condition?

A) Erythema multiforme major secondary to Levetiracetam (Keppra)
B) Toxic Epidermal Necrolysis (TEN) secondary to Carbamazepine
C) Stevens-Johnson syndrome (SJS) secondary to Metformin
D) Acute generalized exanthematous pustulosis secondary to Lisinopril
E) Stevens-Johnson syndrome (SJS) secondary to Atorvastatin

Answer and topic summary

The answer is B) Toxic Epidermal Necrolysis (TEN) secondary to Carbamazepine

The patient’s presentation with widespread epidermal necrosis, mucous membrane involvement, and skin detachment affecting more than 30% of her body surface area is suggestive of Toxic Epidermal Necrolysis (TEN). SJS and TEN are considered a spectrum of the same disease, with SJS affecting less than 10% of body surface area, SJS-TEN overlap affecting 10-30%, and TEN affecting more than 30%. The diagnosis can be confirmed by biopsy (showing necrotic epithelium) if clinical characteristics (eg, target lesions progressing to bullae, ocular and mucous membrane involvement, Nikolsky sign, desquamation in sheets) are inconclusive. Immediate discontinuation of the offending drug is paramount. Patients with TEN often require transfer to a burn unit or an intensive care unit for supportive care. Fluid and electrolyte balance, pain control, and prevention of secondary infections are critical. Immunomodulatory agents like IVIG (intravenous immunoglobulin) or cyclosporine may be considered, but their efficacy is still under debate.

Option A: Erythema multiforme major: Presents with targetoid lesions and is less severe than SJS and TEN. Levetiracetam is not strongly associated with SJS or TEN.

Option D: Acute generalized exanthematous pustulosis is an extensive formation of nonfollicular sterile pustules on erythematous background combined with fever and peripheral blood leukocytosis. This uncommon eruption is most often an allergic reaction because of drugs such as aminopenicillins and sulfonamides inter alia.

A good way to remember the body surface area affected in TEN is “T hree x T en = T hirty percent in Toxic Epidermal Necrolysis

The distinction between SJS, SJS/TEN overlap, and TEN is based on the type of lesions and the amount of the body surface area with blisters and erosions

  • Blisters and erosions cover between 3% and 10% of the body in SJS
  • 11–30% in SJS/TEN overlap
  • over 30% in TEN

Smarty PANCE Content Blueprint Review:

Covered under ⇒  PANCE Blueprint DermatologyDesquamationToxic epidermal necrolysis and Stevens-Johnson syndrome


3. A 32-year-old male presents to his primary care physician complaining of persistent fatigue, unintentional weight loss over the past 3 months, and a painless swelling in the left side of his neck. On examination, there is a non-tender, rubbery 3cm lymphadenopathy in the left cervical region. A subsequent excisional biopsy of the node is performed, and microscopy reveals large atypical cells with abundant cytoplasm and bilobed nuclei, reminiscent of an “owl’s eye” appearance. What is the most likely diagnosis?

A) Acute lymphoblastic leukemia (ALL)
B) Non-Hodgkin’s Lymphoma
C) Burkitt’s lymphoma
D) Hodgkin’s Lymphoma
E) Chronic lymphocytic leukemia (CLL)

Answer and topic summary

The answer is D) Hodgkin’s Lymphoma

The patient’s clinical presentation with painless cervical lymphadenopathy and constitutional symptoms, coupled with the histological finding of large cells bearing the “owl’s eye” appearance (Reed-Sternberg cells), is characteristic of Hodgkin’s Lymphoma. Hodkin’s lymphoma is the most common type of lymphoma and usually presents as a solitary cervical lymph node that has been there for > 30 days. It is commonly manifested with painless cervical adenopathy; there may be splenomegaly or enlargement of other immune tissue, fever, weight loss, fatigue, or night sweats. Upper body lymph nodes are the most common. Diagnosis is based on clinical presentation and confirmed by an excisional biopsy of an involved lymph node. Histology often reveals the pathognomonic Reed-Sternberg cells, which have bilobed nuclei. A chest radiograph should be obtained to search for mediastinal adenopathy. Treatment for Hodgkin’s Lymphoma depends on the stage of the disease and typically involves a combination of chemotherapy, most commonly the ABVD regimen (Adriamycin, Bleomycin, Vinblastine, Dacarbazine). Radiation therapy might be added, especially for localized disease.

Answer Choices Explanations:

A) Acute lymphoblastic leukemia (ALL): Affects mainly children and presents with bone marrow failure symptoms.

B) Non-Hodgkin’s Lymphoma: A group of lymphoid malignancies. Reed-Sternberg cells distinguish Hodgkin’s Lymphoma from Non-Hodgkin’s types.

C) Burkitt’s lymphoma: Fast-growing non-Hodgkin’s lymphoma linked to the Epstein-Barr virus. Exhibits a “starry sky” appearance on microscopy.

E) Chronic lymphocytic leukemia (CLL): A malignancy of mature B cells in older adults. Increased lymphocyte count and smudge cells on a peripheral blood smear.

Smarty PANCE Content Blueprint Review:

Covered under ⇒ PANCE Blueprint HematologyNeoplasms, premalignancies, and malignanciesLymphoma (ReelDx + Lecture)


4. A 65-year-old male presents to your office complaining of fatigue and shortness of breath with exertion. The patient reports minimal cough. On physical exam, you note a thin, barrel-chested man with decreased heart and breath sounds, pursed-lip breathing, end-expiratory wheezing, and scattered rhonchi. Chest X-ray reveals a flattened diaphragm, hyperinflation, and a small, thin-appearing heart. PFTs show a decreased FEV1 / FVC ratio.

A) Asthma
B) Chronic bronchitis
C) Emphysema
D) Pulmonary fibrosis
E) Bronchiectasis

Answer and topic summary

The answer is C. Emphysema

The patient’s presentation of shortness of breath with exertion, barrel-chested appearance, pursed-lip breathing, findings on chest X-ray (e.g., flattened diaphragm, hyperinflation, and a small heart), and pulmonary function test results all align with the diagnosis of emphysema.

Emphysema is a lung disease that causes shortness of breath. It is one of the two main conditions that make up chronic obstructive pulmonary disease (COPD). The other condition is chronic bronchitis.

In emphysema, the air sacs (alveoli) in the lungs are damaged. Over time, the inner walls of the air sacs weaken and rupture — creating larger air spaces instead of many small ones. This reduces the surface area of the lungs and, in turn, the amount of oxygen that reaches your bloodstream.

The main cause of emphysema is cigarette smoking. Other causes include air pollution and chemical fumes. A small percentage of cases are caused by a familial or genetic disorder, alpha-1-antitrypsin deficiency.

  • The body’s natural response to ↓ lung function is chronic hyperventilation = Pink Puffers! CO2 Retainers – the body must increase ventilation to blow off CO2
  • Minimal cough (compared to chronic bronchitis), quiet lungs
  • Minimal sputum (compared to chronic bronchitis)
  • Thin, underweight, and barrel chest

Answer Choices Explanations:

A) Asthma: An obstructive lung disease that is reversible and is marked by bronchoconstriction. It typically presents with episodes of wheezing triggered by factors such as allergens, exercise, or infections.

B) Chronic bronchitis: Defined by a productive cough for 3 consecutive months in 2 successive years. It’s a subtype of chronic obstructive pulmonary disease (COPD), with its main causative factor being smoking. The primary concern is mucus production, as opposed to the alveolar wall destruction seen in emphysema.

D) Pulmonary fibrosis: Represents a set of disorders causing scarring of the lung tissue, leading to a restrictive lung disease pattern. Symptoms might include a dry cough, finger clubbing, and inspiratory crackles upon examination.

E) Bronchiectasis: It is marked by the chronic dilation of bronchi or bronchioles due to repeated infections and inflammation. Patients often have a chronic cough and produce significant amounts of sputum.

Smarty PANCE Content Blueprint Review:

Covered under ⇒ PANCE Blueprint Pulmonary ⇒ Chronic obstructive pulmonary diseases ⇒ Emphysema

Also covered as part of the Internal Medicine EORFamily Medicine EOR, and General Surgery EOR topic lists


5. A 68-year-old woman presents to the emergency department with chest pain that started 5 hours ago. She describes the pain as a heavy pressure radiating to her left arm. It began at rest and has persisted. She has a history of hypertension and diabetes mellitus. An ECG shows ST-segment depressions in leads II, III, and aVF. Troponin I levels are elevated at 0.55 ng/mL (normal: <0.04 ng/mL). Which of the following is the most likely diagnosis?

A) Stable angina
B) Unstable angina
C) Non-ST segment elevation myocardial infarction (NSTEMI)
D) ST-segment elevation myocardial infarction (STEMI)
E) Prinzmetal angina

Answer and topic summary

The answer is C) Non-ST segment elevation myocardial infarction (NSTEMI)

This patient’s presentation with chest pain at rest, ST-segment depressions on ECG, and elevated troponin I levels are consistent with NSTEMI. NSTEMI is characterized by myocardial ischemia severe enough to result in myocyte injury and elevated cardiac biomarkers but not sufficient to cause ST-segment elevation on ECG.

NSTEMI is diagnosed clinically with supporting findings from ECG changes, especially ST-segment depressions or T-wave inversions, and elevated cardiac biomarkers (troponins or CK-MB). Elevated troponins, in particular, differentiate NSTEMI from unstable angina.

Immediate management includes antiplatelet agents (e.g., aspirin, clopidogrel), anticoagulation (e.g., heparin or enoxaparin), and nitrates for symptom relief. A coronary angiography, followed by possible revascularization (percutaneous coronary intervention or coronary artery bypass grafting), might be required based on risk assessment.

Answer Choices Explanations:

A) Stable angina: Characterized by predictable chest pain or discomfort with exertion or stress, which is relieved by rest or nitroglycerin.

B) Unstable angina: Presents as chest pain at rest, new-onset angina, or angina that is more frequent, longer in duration, or not relieved by rest/nitroglycerin. Crucially, cardiac biomarkers (e.g., troponins) remain normal, differentiating it from NSTEMI.

D) ST-segment elevation myocardial infarction (STEMI): Acute myocardial infarction characterized by ST-segment elevation on ECG. Requires immediate reperfusion therapy.

E) Prinzmetal angina: Caused by coronary artery spasm leading to transient ST-segment elevation. Pain typically occurs at rest, often in the early morning hours.

Smarty PANCE Content Blueprint Review:

PANCE Blueprint Cardiology => Coronary Heart Disease (PEARLS) => Non-ST-Segment Elevation MI (NSTEMI) ReelDx


This podcast is available on every device!

You can download and listen to past FREE episodes here, on iTunes, Spotify, Google Podcasts, Stitcher, Amazon Music, and all podcasting apps.

Download Interactive Content Blueprint Checklists for the PANCE, PANRE, EOR, and PANRE-LA

Interactive Content Blueprints for the PANCE PANRE and PANRE-LA

Follow this link to download your FREE copy of the PANCE/PANRE/EOR Content Blueprint Checklists.

Print it up and start crossing out the topics you understand, marking the ones you don’t, and making notes of key terms you should remember. The PDF version is interactive and linked directly to the individual lessons on Smarty PANCE.

Smarty PANCE is not sponsored or endorsed by, or affiliated with, the National Commission on Certification of Physician Assistants.

The post Podcast Episode 107: This vs. That – PANCE Blueprint Comparisons You Need to Know (Part 1) appeared first on The Audio PANCE and PANRE.

]]>
Listen to Podcast Episode 107: This vs. That – PANCE Blueprint Comparisons You Need to Know (Episode 1) In today’s session, we will be discussing five questions related to PANCE/PANRE Blueprint topics. These questions will cover similar presentations a... In today’s session, we will be discussing five questions related to PANCE/PANRE Blueprint topics. These questions will cover similar presentations and crucial comparisons that are important for you to know. These topics are often used by PANCE/PANRE test question writers, so it’s essential to learn how to differentiate between them. This is the first part (episode 1) of a series.
If you can’t see the audio player, click here to listen to the full episode.

Links from today’s episode:

* Sign up for our new PANCE and PANRE Test-Taking Masterclass.
* Check out my first blog in our “This vs. That” Blueprint series: The PANCE Blueprint Showdown: Crohn’s Disease vs. Ulcerative Colitis.
* Want a question of the day that covers the Blueprint – ALL of it? Sign up for the Entire Blueprint Email Series.
* Follow Smarty PANCE and The Daily PANCE Blueprint on Instagram and Facebook for more daily questions.
* Join the Smarty PANCE Member’s Community, then sign up for a study group to get updates about upcoming webinars.

I hope you enjoy this free audio component of the examination portion of this site. Smarty PANCE includes over 2,000 interactive board review questions, along with flashcards, ReelDx cases, integrated Picmonics, and lessons covering every blueprint topic available to all Smarty PANCE members.

* You can download and listen to past FREE episodes here, on iTunes, Spotify, Google Podcasts, Stitcher, Amazon Music, and all podcasting apps.
* You can listen to all the latest episodes, take interactive quizzes, and download more resources on each episode page.

Interactive exam to complement today’s podcast
1. A 32-year-old woman presents with a 6-month history of loose bowel movements, approximately eight per day. Blood has been present in many of them. She has lost 30 pounds. For the past 6 weeks, she has had intermittent fever. She has had no previous gastrointestinal (GI) problems, and there is no family history of GI problems. On examination,]]>
The Physician Assistant Life | Smarty PANCE full 30:56 591
Podcast Episode 106: Ten PANCE, PANRE, and Rotation Questions + Review of Adrenal Insufficiency http://podcast.thepalife.com/episode-106/ Wed, 14 Jun 2023 09:00:45 +0000 https://podcast.thepalife.com/?p=581 Listen to Podcast Episode 106: Ten PANCE, PANRE, and Rotation Review Questions + Review of Adrenal Insufficiency If you can’t see the audio player, click here to listen to the full episode. Welcome to episode 106 of the Audio PANCE and PANRE Physician Assistant/Associate Board Review Podcast. Join me today as we cover ten board […]

The post Podcast Episode 106: Ten PANCE, PANRE, and Rotation Questions + Review of Adrenal Insufficiency appeared first on The Audio PANCE and PANRE.

]]>
Listen to Podcast Episode 106: Ten PANCE, PANRE, and Rotation Review Questions + Review of Adrenal Insufficiency

If you can’t see the audio player, click here to listen to the full episode.

Podcast Episode 106 - The Audio PANCE and PANRE Board Review PodcastWelcome to episode 106 of the Audio PANCE and PANRE Physician Assistant/Associate Board Review Podcast.

Join me today as we cover ten board review questions for your PANCE, PANRE, EOR, and EOC exams.

Links from today’s episode:

I hope you enjoy this free audio component of the examination portion of this site. Smarty PANCE includes over 2,000 interactive board review questions, along with flashcards, ReelDx cases, integrated Picmonics, and lessons covering every blueprint topic available to all Smarty PANCE members.

Interactive exam to complement today’s podcast

1. A 22-year-old female with an unknown past medical history presents to the ER with a prolonged seizure lasting more than 5 minutes per EMS. Her airway was supported, and IV access was obtained en route to the ER. Her blood glucose is 120. Her vitals are stable, and laboratory studies are relatively unremarkable. Which of the following would be an appropriate medication to give to this patient?

A. Calcium gluconate
B. Propofol infusion
C. Lorazepam
D. Clonidine
E. Narcan

Answer and topic summary

The answer is C. Lorazepam

The patient has status epilepticus, which is defined as >5 minutes of continuous seizures or > 2 discrete seizures between which there is incomplete recovery of consciousness. Most episodes of status epilepticus in adults are due to a brain lesion or a toxic/metabolic disturbance (e.g., alcohol withdrawal, hypoglycemia, etc). Immediate management of status epilepticus includes stabilizing the airway, placing pulse oximetry and cardiorespiratory monitors, establishing IV access, and getting a glucose level. The first-line pharmacological agent given is a benzodiazepine (e.g., lorazepam or diazepam). Keppra, valproate, or other anti-seizure medication can be given as well.

Smarty PANCE Content Blueprint Review:

Covered under ⇒ PANCE Blueprint NeurologySeizure disordersStatus epilepticus

Also covered as part of the Internal Medicine EOR and Emergency Medicine EOR topic list


2. A 41-year-old male presents to the ER after a construction accident that left him with severe right eye pain and decreased visual acuity. On a physical exam, you notice a teardrop-shaped pupil. Which of the following is the most likely diagnosis?

A. Globe rupture
B. Acute angle-closure glaucoma
C. Retinal detachment
D. Corneal laceration
E. Metallic foreign body

Answer and topic summary

The answer is A. Globe rupture

Mechanical globe injuries occur when there is a laceration or full-thickness rupture through the cornea and/or sclera. Globe rupture (also called an open globe) follows blunt eye injury (e.g., motor vehicle crash, assault, thrown ball, etc). Globe lacerations occur after trauma from a sharp-penetrating object (e.g., knife or high-velocity projectile). PE signs include subconjunctival hemorrhage, irregularly-shaped pupil, hyphema (anterior chamber bleeding), decreased visual acuity, and limited EOM. This is an emergency and a patient should see an ophthalmologist immediately.

Smarty PANCE Content Blueprint Review:

Covered under ⇒ PANCE Blueprint EENTTraumatic disordersGlobe rupture


3. A 45-year-old male veteran presents to your clinic complaining of the following for 4 months: problems with concentration, difficulty staying asleep, persistent negative state, inability to remember events that happened while he was in Iraq, feelings of detachment from others, avoidance of other veterans, and distress when seeing violence on the TV. He denies hallucinations. He has been a veteran for 5 years. Which of the following is the most likely diagnosis?

A. Acute stress disorder
B. Cyclothymic disorder
C. Brief psychotic episode
D. Derealization/depersonalization disorder
E. Post-traumatic stress disorder

Answer and topic summary

The answer is E. Post-traumatic stress disorder

Post-traumatic stress disorder is a complex disorder defined by the behavioral, somatic, cognitive, and emotional effects of trauma. The most common causes are sexual relationship violence (33%), death or traumatic event of a loved one (30%), interpersonal violence like physical assault (12%), and participation in organized violence like combat (11%). The DSM-V criteria for PTSD is:

Exposure to a traumatic event(s). And then for ≥1 month of each of these that started/worsened after a traumatic event and are associated with the traumatic event:

1) Intrusion symptoms

2) Avoidance of stimuli

3) Alterations in cognitions and mood

4) Alterations in arousal/reactivity

Treatment is SSRIs + counseling/psychotherapy. About 50% of patients will recover in 3 months with treatment.

Smarty PANCE Content Blueprint Review:

Covered under ⇒ PANCE Blueprint PsychiatryTrauma and stressor-related disordersPost-traumatic stress disorder

Also covered in Emergency Medicine EOR, and Family Medicine EOR Blueprint


4. Which of the following clinical findings would you likely expect in a patient with primary adrenal insufficiency?

A. Hypernatremia, hypokalemia, tired
B. Moon facies, depression, bruising
C. Polyuria, polydipsia, intense thirst
D. Fatigue, hyponatremia, hyperpigmentation
E. Hypothermia, bradycardia, goiter

Answer and topic summary

The answer is d. Fatigue, hyponatremia, hyperpigmentation

Adrenal insufficiency is defined as insufficient production of hormones by the adrenal glands. Remember, the MOST common cause of adrenal insufficiency overall is the sudden withdrawal of exogenous steroids. The most common primary cause of adrenal insufficiency is autoimmune adrenalitis (aka Addison disease, aka 21-hydroxylase antibodies against the adrenal gland). Patients will have symptoms and signs of glucocorticoid & mineralocorticoid deficiency, such as fatigue (MC), weight loss, nausea, vomiting, hyponatremia, hyperkalemia, skin hyperpigmentation (due to increased production of proopiomelanocortin prohormone), hypotension, and salt craving. Remember, secondary adrenal insufficiency does NOT lead to hyperpigmentation, only primary adrenal insufficiency (this is commonly asked in tests). Diagnosis of adrenal insufficiency is made by serum cortisol level and then a cosyntropin stimulation test. Treatment is glucocorticoids +/- mineralocorticoids.

Smarty PANCE Content Blueprint Review:

Covered under ⇒ PANCE Blueprint EndocrinologyAdrenal DisordersPrimary adrenal insufficiency

Also covered as part of the Family Medicine EOR, Internal Medicine EOR, and Emergency Medicine EOR topic list


5. A 42-year-old male presents with right upper extremity numbness, dysesthesia, weakness, and arm pain with exertion. The symptoms are aggravated by activity and reaching overhead. Which of the following is the most likely diagnosis?

A. Thoracic outlet syndrome
B. Median nerve compression
C. Biceps tendon rupture
D. Carpal tunnel syndrome
E. Subclavian steal syndrome

Answer and topic summary

The answer is A. Thoracic outlet syndrome

Thoracic outlet syndrome (TOS) refers to a constellation of signs & symptoms that occur due to compression of the neurovascular bundle in the space of the thoracic outlet. Causes include trauma, repetitive injuries, anatomical defects, and pregnancy. The clinical presentation depends on what is being compressed (i.e., nerve, vein, and/or artery), but may include the following: pain, numbness, weakness, muscle weakness, fatigue, swelling, or coldness. Often these symptoms are aggravated by activity or raising the arm overhead. Diagnostic tests include ultrasound, x-ray, CT scan, EMG, MRI, etc. Treatment options depend on what type of TOS the patient has (e.g., physical therapy for neurogenic TOS).

Smarty PANCE Content Blueprint Review:

Covered under ⇒ PANCE Blueprint MusculoskeletalSpinal DisordersThoracic outlet syndrome


6. A 34-year-old female is diagnosed with immune thrombocytopenic purpura. What is NOT a known cause or risk factor of ITP?

A. Hepatitis C
B. Systemic lupus erythematosus
C. HIV
D. COVID-19
E. Warfarin

Answer and topic summary

The answer is E. Warfarin

Immune thrombocytopenic purpura (ITP) is acquired thrombocytopenia caused by autoantibodies against platelet antigens. Primary ITP is specifically due to autoimmune mechanisms, whereas secondary ITP is associated with other conditions or drugs. Some examples of causes of secondary ITP include systemic lupus erythematosus (SLE), HIV infection, hepatitis C infection, COVID-19, and thyroid dysfunction. There are also many drugs that can lead to ITP, but warfarin is not known to be one of them. Clinical symptoms include petechiae, purpura, easy bleeding, bruising. Obviously, platelets will be low. Work-up for ITP should include HIV and HCV testing, TSH, ANA, coagulation studies, and peripheral blood smear. A bone marrow biopsy can be done to rule out something more pathological. Treatment includes steroids (high-dose) and IVIG. Definitive treatment is splenectomy.

Smarty PANCE Content Blueprint Review:

Covered under ⇒ PANCE Blueprint HematologyCoagulation DisordersThrombocytopeniaIdiopathic thrombocytopenic purpura

Also covered as part of the Internal Medicine EOR topic list


7. A 72-year-old African American male presents to the clinic complaining of weight loss, fatigue, and back pain. During the physical exam, you palpate an enlarged, irregular, nodular prostate. Which of the following is the most important risk factor for the development of this most likely diagnosis?

A. Genetic factors
B. Ethnicity
C. Increasing age
D. Obesity
E. High meat diet

Answer and topic summary

The answer is C. Increasing age

Prostate cancer is the most common cancer among men (excluding skin cancer) — as about 1 in 8 men will get it. The most important risk factor for the development of prostate cancer is increasing age. Other risk factors include ethnicity, genetic factors, obesity, tobacco, and a diet high in animal fat. Clinical manifestations may include non-specific urinary symptoms (frequency, urgency, etc), hematuria, bone pain, elevated PSA, and an indurated/asymmetrical prostate with nodules on digital rectal exam. Diagnosis is made by a transrectal biopsy of the prostate.

Smarty PANCE Content Blueprint Review:

Covered under ⇒ PANCE Blueprint GenitourinaryNeoplasms of the Genitourinary SystemProstate cancer

Also found in Internal Medicine EOR PAEA blueprint


8. A 52-year-old female with a history of previous abdominal surgeries presents to the ER with nausea, emesis, abdominal pain, and inability to pass gas. On physical exam, you notice abdominal distention and auscultate high-pitched tinkering sounds in the upper abdominal quadrants. Which of the following is the most likely diagnosis?

A. Mesenteric ischemia
B. Diverticulitis
C. Gastric ulcer perforation
D. Small bowel obstruction
E. Ischemic colitis

Answer and topic summary

The answer is D. Small bowel obstruction

A bowel obstruction happens when the normal flow of the GI tract is interrupted. The most common cause of small bowel obstruction is intraperitoneal adhesions (s/t previous surgeries). Other causes include hernias or neoplasms. Clinical features include nausea, vomiting, crampy abdominal pain, and obstipation (inability to pass stool). On physical exam you may note abdominal distention, tinkling high-pitch sounds on auscultation, and hyperresonance to percussion. Initial diagnostic modalities include an abdominal x-ray and CT with contrast. These patients need to be admitted for further evaluation and management. Surgery should be consulted.

Smarty PANCE Content Blueprint Review:

Covered under ⇒ PANCE Blueprint GI and NutritionDiseases of the Small IntestineSmall bowel obstruction

Also covered as part of the Family Medicine EOR, Emergency Medicine EOR, and General Surgery EOR topic list


9. Which of the following is the most common cause of acute cor pulmonale?

A. Cardiac tamponade
B. Pulmonary embolism
C. Mitral regurgitation
D. Congestive heart failure
E. Portopulmonary syndrome

Answer and topic summary

The answer is B. Pulmonary embolism

The most common cause of acute cor pulmonale is a massive pulmonary embolism (PE). Cor pulmonale is defined by RV failure/enlargement due to a pulmonary issue. The reason a PE leads to acute right heart failure is because a PE is a mechanical obstruction that increases vascular resistance in lungs and increases RV afterload. The RV can’t unload sufficiently and this results in dilation of the RV. The dilated RV impedes on the LV, leading to decreased LV output and decreased supply to the coronary arteries. It also causes RV wall tension/pressure to build and thus coronary perfusion is impeded to the right heart, further leading to ischemia. Also, the dilation of the RV can lead to tricuspid regurgitation, which further reduces blood flow. As the RV becomes ischemic, it can’t contract as well and this further decreases RV output and LV output — leading to a dangerous cycle toward shock. You can risk stratify the degree of RV compromise by getting an echocardiogram, BNP, & troponin.

Smarty PANCE Content Blueprint Review:

Covered under ⇒ PANCE Blueprint PulmonaryPulmonary Circulation Cor pulmonale


10. A 25-year-old female with no past medical history presents with nausea, emesis, and pelvic pain. She is normotensive, afebrile, and slightly tachycardic (HR 102 bpm). On physical exam, you palpate an ovarian/pelvic mass. Beta-hCG is negative. The pelvic US is pending. Which of the following is the most likely diagnosis?

A. Ectopic pregnancy
B. Appendicitis
C. Pelvic inflammatory disorder
D. Ovarian torsion
E. Placenta abruption

Answer and topic summary

D. Ovarian torsion

Ovarian torsion refers to the rotation of the ovary, often leading to complete or partial obstruction of blood supply. The most common predisposing factor is an ovarian mass. Presenting features include pelvic pain (90%), nausea, emesis, and fever. On physical exam they may have pelvic and/or abdominal tenderness. A pelvic ultrasound is the mainstay of evaluation when ovarian torsion is suspected. You can get a beta-hCG to rule out an ectopic pregnancy. The definitive diagnosis is made by directly visualizing a rotated ovary at the time of surgical evaluation. Surgery and OB-GYN should be consulted for further management.

Smarty PANCE Content Blueprint Review:

Covered under ⇒ PANCE Reproductive System BlueprintOvarian Torsion

Also covered as part of the Women’s Health PAEA EOR topic lists


This podcast is available on every device!

You can download and listen to past FREE episodes here, on iTunes, Spotify, Google Podcasts, Stitcher, Amazon Music, and all podcasting apps.

Download Interactive Content Blueprint Checklists for the PANCE, PANRE, EOR, and PANRE-LA

Interactive Content Blueprints for the PANCE PANRE and PANRE-LA

Follow this link to download your FREE copy of the PANCE/PANRE/EOR Content Blueprint Checklists.

Print it up and start crossing out the topics you understand, marking the ones you don’t, and making notes of key terms you should remember. The PDF version is interactive and linked directly to the individual lessons on Smarty PANCE.

Smarty PANCE is not sponsored or endorsed by, or affiliated with, the National Commission on Certification of Physician Assistants.

The post Podcast Episode 106: Ten PANCE, PANRE, and Rotation Questions + Review of Adrenal Insufficiency appeared first on The Audio PANCE and PANRE.

]]>
Listen to Podcast Episode 106: Ten PANCE, PANRE, and Rotation Review Questions + Review of Adrenal Insufficiency If you can’t see the audio player, click here to listen to the full episode. Welcome to episode 106 of the Audio PANCE and PANRE Physician ... If you can’t see the audio player, click here to listen to the full episode.
Welcome to episode 106 of the Audio PANCE and PANRE Physician Assistant/Associate Board Review Podcast.
Join me today as we cover ten board review questions for your PANCE, PANRE, EOR, and EOC exams.
Links from today’s episode:

* Sign up for our new PANCE and PANRE Test Taking Masterclass
* Sign up for the Entire Blueprint Email Series
* Follow Smarty PANCE and The Daily PANCE Blueprint on Instagram and Facebook
* Review adrenal insufficiency and adrenal disorders
* Join the Smarty PANCE Member’s Community, then sign up for a study group to get updates about upcoming webinars.

I hope you enjoy this free audio component of the examination portion of this site. Smarty PANCE includes over 2,000 interactive board review questions, along with flashcards, ReelDx cases, integrated Picmonics, and lessons covering every blueprint topic available to all Smarty PANCE members.

* You can download and listen to past FREE episodes here, on iTunes, Spotify, Google Podcasts, Stitcher, Amazon Music, and all podcasting apps
* You can listen to all the latest episodes, take interactive quizzes, and download more resources on each episode page.

Interactive exam to complement today’s podcast
1. A 22-year-old female with an unknown past medical history presents to the ER with a prolonged seizure lasting more than 5 minutes per EMS. Her airway was supported, and IV access was obtained en route to the ER. Her blood glucose is 120. Her vitals are stable, and laboratory studies are relatively unremarkable. Which of the following would be an appropriate medication to give to this patient?
A. Calcium gluconate
B. Propofol infusion
C. Lorazepam
D. Clonidine
E. Narcan
Smarty PANCE Content Blueprint Review:
Covered under ⇒ PANCE Blueprint Neurology full 28:55 581
Podcast Episode 105: Ten PANCE, PANRE, and Rotation Review Questions http://podcast.thepalife.com/podcast-episode-105-ten-pance-panre-and-rotation-review-questions/ Mon, 08 May 2023 09:00:03 +0000 https://podcast.thepalife.com/?p=573 Listen to Podcast Episode 105: Ten PANCE, PANRE, and Rotation Review Questions If you can’t see the audio player, click here to listen to the full episode. Welcome to episode 105 of the Audio PANCE and PANRE Physician Assistant/Associate Board Review Podcast. Join me today as we cover ten board review questions for your PANCE, […]

The post Podcast Episode 105: Ten PANCE, PANRE, and Rotation Review Questions appeared first on The Audio PANCE and PANRE.

]]> Listen to Podcast Episode 105: Ten PANCE, PANRE, and Rotation Review Questions

If you can’t see the audio player, click here to listen to the full episode.

Podcast Episode 105 - The Audio PANCE and PANRE Board Review PodcastWelcome to episode 105 of the Audio PANCE and PANRE Physician Assistant/Associate Board Review Podcast.

Join me today as we cover ten board review questions for your PANCE, PANRE, EOR, and EOC exams.

Links from today’s episode:

I hope you enjoy this free audio component of the examination portion of this site. Smarty PANCE includes over 2,000 interactive board review questions, along with flashcards, ReelDx cases, integrated Picmonics, and lessons covering every blueprint topic available to all Smarty PANCE members.

Interactive exam to complement today’s podcast

1. A 68-year-old male farmer presents with a flesh-colored papule with a rolled border located on the right side of his forehead. As you examine the lesion closely, you notice something else about the lesion. Which of the following physical exam findings would make you more suspicious of malignancy?

A. Telangiectasia
B. Nikolsky sign
C. Hypopigmentation
D. Tenderness to palpation
E. Central umbilication

Answer and topic summary

The answer is A. Telangiectasia

The patient has basal cell carcinoma, which is a skin cancer with low metastatic potential. It commonly occurs on the face (70% of the time). There are different types of BCC (nodular vs. superficial vs. infiltrative). However, nodular is the most common (80%) and typical characteristics you may see include a papule with a rolled border, pearly-like look, flesh-colored, and telangiectasia. Risk factors for BCC include UV radiation, certain genes, inherited disorders, etc.

Smarty PANCE Content Blueprint Review:

Covered under ⇒ PANCE Blueprint DermatologyDermatologic NeoplasmsBasal cell carcinoma

Also covered as part of the General Surgery EOR and Family Medicine EOR topic list

2. Which of the following is the most common type of elder abuse?

A. Neglect
B. Emotional abuse
C. Physical abuse
D. Sexual abuse
E. Financial exploitation

Answer and topic summary

The answer is A. Neglect

The most common type of elder abuse is neglect, which refers to the failure of a trusted person to protect an older person from harm or provide for their needs. Self-neglect is also common, which is when an older person can’t proide their own care. Some warning signs include bruising, lacerations, skin tears, spiral fractures, malnutrition, pressure ulcers, and dehydration. If you even suspect an older adult is being neglected, you should report this immediately to adult protective services (or similar agencies) and treat the medical complications immediately.

Smarty PANCE Content Blueprint Review:

Covered under ⇒ PANCE Blueprint PsychiatryAbuse and NeglectChild/elder abuse

3. Which of the following is the most common bacterial cause of a hordeolum?

A. Streptococcus pyogenes
B. Staphylococcus aureus
C. Clostridium perfringens
D. Propionibacterium species
E. Enterobacter aerogenes

Answer and topic summary

The answer is B. Staphylococcus aureus

A hordeolum (stye) is an abscess of the eyelid that is acute. It will usually present with pain and swelling of the eyelid. The most common pathogen indicated is Staphylococcus aureus. Risk factors include dirty eye makeup, rosacea, and seborrheic dermatitis. A way to differentiate hordeolums from chalazions is that Hordeolums are Hot (aka warm/tender).

Treatment may include warm compresses (10 min up to 5 times daily) plus massaging the eyelid to aid in drainage.

Smarty PANCE Content Blueprint Review:

Covered under ⇒ PANCE Blueprint EENTDisorders of the Eye Lid disordersHordeolum

Also covered as part of the Family Medicine EOR topic list

4. A 19-year-old female presents to the clinic with fatigue, weight loss, arthralgia, and an erythematous rash in a malar distribution over the cheeks and nose (that spares the nasolabial folds). Which of the following is the most common cardiac manifestation of the patient’s likely diagnosis?

A. Third-degree AV block
B. Mitral regurgitation
C. Myocarditis
D. Pericarditis
E. Bradydysrhythmias

Answer and topic summary

The answer is D. Pericarditis

Systemic lupus erythematosus (SLE) is a chronic autoimmune disease that can impact almost any organ. It’s important to be aware of how SLE clinically presents in order to get to a diagnosis and treatment.

The most common clinical features overall are fatigue (80-100%), arthralgia/arthritis (>90%), fever (>50%), and Raynaud phenomenon (~50%). You may also see the classic malar rash that spares nasolabial folds.

Cardiac complications are common (50%) in SLE, and they lead to morbidity and mortality. The most common cardiac manifestation is pericarditis. Other cardiac issues these patients may have include pericardial effusion, myocarditis (rare), and valvular dysfunction.

Smarty PANCE Content Blueprint Review:

Covered under ⇒ PANCE Blueprint MusculoskeletalRheumatologic DisordersSystemic lupus erythematosus

Also covered as part of the Internal Medicine EOR and Family Medicine EOR topic list

5. Which of the following is the most common and important risk factor for aortic dissection?

A. Bicuspid aortic valve
B. Connective tissue disorder
C. Turner’s syndrome
D. Systemic hypertension
E. Aortic surgery

Answer and topic summary

The answer is D. Systemic hypertension

Aortic dissection is a life-threatening emergency and is defined by a tear of the inner layer of the aorta. On an exam, you may be asked about the classifications:

Standford Classification: Stanford A is ascending aorta and Standford B is descending aorta

DeBakey Classifications: type I is Both (ascending + descending aorta), type II is Ascending aorta, and III is Descending aorta. (Hint: It spells out BAD.)

The most important & common risk factor is systemic hypertension. Known causes of acute, severe increases in blood pressure include weight lifting, energy drinks, and cocaine. The classic presentation is “tearing and ripping chest pain with pulse deficits.” Treatment is lowering the BP via IV beta-blockers for both Type A and B and then surgery (for Type A).

Smarty PANCE Content Blueprint Review:

Covered under ⇒ PANCE Blueprint CardiologyVascular DiseaseAortic aneurysm/dissection

6. Which of the following is characterized by a rash that has a “slapped cheek” appearance at first and then progresses into a maculopapular rash on the arms and trunk in a reticular pattern?

A. Roseola infantum
B. Rubella
C. Measles
D. Erythema infectiosum
E. Kawasaki disease

Answer and topic summary

The answer is D. Erythema infectiosum

Erythema infectiosum is a condition characterized by a fever with a rash. It is caused by parvovirus B19 infection. The classic presentation is a “slapped cheek” rash on the face and an erythematous, lacy reticular rash on the trunk and extremities. Some call it the “fifth disease.” It is a common childhood viral exanthem. Diagnosis is usually clinical (serology can be done though). The disease is self-limiting and no specific therapy is warranted (most of the time).

Smarty PANCE Content Blueprint Review:

Covered under ⇒ PANCE Blueprint Infectious DiseaseViral Infectious DiseaseErythema infectiosum

Also covered as part of the Pediatric Rotation EOR topic list

7. Which of the following is the most common cause of generalized musculoskeletal pain in young adult females?

A. Multiple sclerosis
B. Rheumatoid arthritis
C. Polymyalgia rheumatica
D. Menstruation
E. Fibromyalgia

Answer and topic summary

The answer is E. Fibromyalgia

Fibromyalgia is the most common cause of generalized muscle/bone pain in young adult females. Patients with fibromyalgia also usually have fatigue, cognitive fog, sleep disturbances, morning stiffness, headaches, paresthesias, and depression. On a physical exam, you will see tenderness to palpation at multiple sites. Initial treatment includes patient education, aerobic exercise, management of concurrent issues (e.g., sleep and psychiatric), & tricyclic antidepressants (e.g., amitriptyline 10 mg).

Smarty PANCE Content Blueprint Review:

Covered under ⇒ PANCE Blueprint MusculoskeletalRheumatologic DisordersFibromyalgia

Also covered as part of the Internal Medicine EOR and Family Medicine EOR topic list

8. Which of the following is the best first-line treatment for a post-menopausal woman with osteoporosis and no pathological fractures?

A. Bisphosphonates
B. Parathyroid hormone analogs
C. RANKL inhibitors
D. Selective estrogen receptor modulators
E.Calcitonin

Answer and topic summary

The answer is A. Bisphosphonates

Osteoporosis is characterized by low bone mass and skeletal fragility leading to decreased bone strength. There are no clinical symptoms of osteoporosis until a patient has an actual fracture. Vertebral fractures in particular, are the MOST common clinical manifestation of osteoporosis.

Diagnosis is made by a DEXA scan (T-score < -2.5 standard deviations) or a fragility fracture.

Lifestyle changes that can be made include exercise and cessation of smoking. The first-line pharmacological agent is oral bisphosphonates (e.g., alendronate or risedronate).

Remember bisphosphonates should be administered first thing in the AM and >30 min before the first food. Patients should stay upright for >30 minutes.

Smarty PANCE Content Blueprint Review:

Covered under ⇒ PANCE Blueprint MusculoskeletalRheumatologic DisordersOsteoporosis

Also covered as part of the Family Medicine EOR topic list

9. A 72-year-old female with sick sinus syndrome (s/p pacemaker) presents to your clinic with fever and chills. On a physical exam, you appreciate a new murmur located at the left 5th intercostal space at the midclavicular line. You also notice petechiae on the patient’s conjunctivae. Which of the following is the most likely diagnosis?

A. Bacterial pneumonia
B. Pericarditis
C. Rheumatic fever
D. Infective endocarditis
E. Pleural effusion

Answer and topic summary

The answer is D. Infective endocarditis

The patient has infective endocarditis (IE), which is an infection of the endocardial surface of the heart. Risk factors include pre-existing valvular or congenital heart disease, IVDU, indwelling cardiac devices, IV catheters, or recent dental/surgical procedures. The most common symptoms include fever (90%), new murmur (85%), and petechiae (20-50%). Other textbook findings include Janeway lesions (non-tender macules on palms/soles), Osler nodes (tender nodules on pads of fingers/toes), and Roth spots (hemorrhagic lesions of the retina). The best diagnostic test is a transesophageal echocardiogram. The modified Duke criteria can be used as a diagnostic guide but must be interpreted in light of pretest probability. At least 3 sets of blood cultures should be obtained. Treatment includes IV antibiotics and removal of any infected device.

Smarty PANCE Content Blueprint Review:

Covered under ⇒ PANCE Blueprint CardiologyTraumatic, infectious, and inflammatory heart conditionsAcute and subacute bacterial endocarditis

Also covered as part of the Emergency Medicine EOR, Internal Medicine EOR, and Family Medicine EOR topic list

10. Which of the following is not considered diagnostic for diabetes mellitus?

A. Random plasma glucose > 200 mg/dL with symptoms of hyperglycemia
B. A1c > 6.5
C. Fasting plasma glucose > 126 mg/dL on at least two separate occasions
D. Plasma glucose > 200 mg/dL measured after a glucose tolerance test
E. All of the above are diagnostic

Answer and topic summary

The answer is E. All of the above are diagnostic 

Type 2 diabetes mellitus is a very prevalent, serious disease that impacts about 9% of adults in the United States. Risk factors include family history, obesity, sedentary lifestyle, smoking, and poor diet habits. Initial symptoms include polyuria, polydipsia, polyphagia, and blurred vision. Diagnosis is made by one of the following answer choices seen on the first slide. First-line therapies include metformin and lifestyle changes (i.e., weight loss + exercise). Early combination therapy can be considered at treatment initiation to extend the time to treatment failure. It is VERY important to reach tight glycemic control as soon as possible in diabetic patients. Complications of diabetes include microvascular (retinopathy, nephropathy, neuropathy) and macrovascular (stroke, myocardial infarction).

Smarty PANCE Content Blueprint Review:

Covered under ⇒ PANCE Blueprint EndocrinologyDiabetes MellitusDiabetes Mellitus Type 2

Also covered as part of the Family Medicine EOR and Emergency Medicine EOR topic list

This podcast is available on every device!

You can download and listen to past FREE episodes here, on iTunes, Spotify, Google Podcasts, Stitcher, Amazon Music, and all podcasting apps.

Download Interactive Content Blueprint Checklists for the PANCE, PANRE, EOR, and PANRE-LA

Interactive Content Blueprints for the PANCE PANRE and PANRE-LA

Follow this link to download your FREE copy of the PANCE/PANRE/EOR Content Blueprint Checklists.

Print it up and start crossing out the topics you understand, marking the ones you don’t, and making notes of key terms you should remember. The PDF version is interactive and linked directly to the individual lessons on Smarty PANCE.

Smarty PANCE is not sponsored or endorsed by, or affiliated with, the National Commission on Certification of Physician Assistants.

The post Podcast Episode 105: Ten PANCE, PANRE, and Rotation Review Questions appeared first on The Audio PANCE and PANRE.

]]>
Listen to Podcast Episode 105: Ten PANCE, PANRE, and Rotation Review Questions If you can’t see the audio player, click here to listen to the full episode. Welcome to episode 105 of the Audio PANCE and PANRE Physician Assistant/Associate Board Review P... If you can’t see the audio player, click here to listen to the full episode.
Welcome to episode 105 of the Audio PANCE and PANRE Physician Assistant/Associate Board Review Podcast.
Join me today as we cover ten board review questions for your PANCE, PANRE, EOR, and EOC exams.
Links from today’s episode:

* Sign up for the Entire Blueprint Email Series
* Follow Smarty PANCE and The Daily PANCE Blueprint on Instagram
* Follow Smarty PANCE and The Daily PANCE Blueprint on Facebook
* Review systemic lupus erythematosus (SLE)
* Review the diabetes diagnostic guidelines
* Review gestational diabetes screening guidelines
* Review basal cell carcinoma and our comparison tables of the Blueprint dermatologic neoplasms
* Join the Smarty PANCE Member’s Community, then sign up for a study group to get updates about upcoming webinars.

I hope you enjoy this free audio component of the examination portion of this site. Smarty PANCE includes over 2,000 interactive board review questions, along with flashcards, ReelDx cases, integrated Picmonics, and lessons covering every blueprint topic available to all Smarty PANCE members.

* You can download and listen to past FREE episodes here, on iTunes, Spotify, Google Podcasts, Stitcher, Amazon Music, and all podcasting apps
* You can listen to all the latest episodes, take interactive quizzes, and download more resources on each episode page.

Interactive exam to complement today’s podcast
1. A 68-year-old male farmer presents with a flesh-colored papule with a rolled border located on the right side of his forehead. As you examine the lesion closely, you notice something else about the lesion. Which of the following physical exam findings would make you more suspicious of malignancy?
A. Telangiectasia
B. Nikolsky sign
C. Hypopigmentation
D. Tenderness to palpation
E.]]>
The Physician Assistant Life | Smarty PANCE full 25:57 573 Podcast Episode 104: Ten PANCE, PANRE, and Rotation Review Questions http://podcast.thepalife.com/episode-104/ Mon, 20 Mar 2023 17:04:50 +0000 https://podcast.thepalife.com/?p=568 Listen to Podcast Episode 104: Ten PANCE, PANRE, and Rotation Review Questions If you can’t see the audio player, click here to listen to the full episode. Welcome to episode 104 of the Audio PANCE and PANRE Physician Assistant/Associate Board Review Podcast. Join me today as we cover ten board review questions for your PANCE, […]

The post Podcast Episode 104: Ten PANCE, PANRE, and Rotation Review Questions appeared first on The Audio PANCE and PANRE.

]]>
Listen to Podcast Episode 104: Ten PANCE, PANRE, and Rotation Review Questions

If you can’t see the audio player, click here to listen to the full episode.

Welcome to episode 104 of the Audio PANCE and PANRE Physician Assistant/Associate Board Review Podcast.

Join me today as we cover ten board review questions for your PANCE, PANRE, EOR, and EOC exams.

Links from today’s episode:

I hope you enjoy this free audio component of the examination portion of this site. Smarty PANCE includes over 2,000 interactive board review questions, along with flashcards, ReelDx cases, integrated Picmonics, and lessons covering every blueprint topic available to all Smarty PANCE members.

Smarty PANCE is not sponsored or endorsed by, or affiliated with, the National Commission on Certification of Physician Assistants.

Interactive exam to complement today’s podcast

When is screening for gestational diabetes done?

A. 16 weeks
B. 22 weeks
C. 24 weeks
D. 32 weeks
E. 34 weeks

Answer and topic summary

The answer is C. 24 weeks

Prenatal care is extremely important. Screening for gestational diabetes is routinely done in pregnant patients at 24 weeks of gestation (typically until 28 weeks). Pregnancy is associated with insulin resistance, mostly because of the placenta’s secretion of human placental lactogen. There are bad consequences of gestational diabetes, so it is critical it is diagnosed and treated adequately.

The initial test is a one-hour 50-gram oral glucose tolerance test (GTT). A positive test >135 mg/dL. If a patient tests positive, they need to undergo the second test, which is a three-hour 100 mg oral GTT. The cut-offs are debated, but generally, the following are positive results: fasting > 95 mg/dL, 1 hour>180 mg/dL, 2 hours>155 mg/dL, 3 hours>140 mg/dL.

Smarty PANCE Content Blueprint Review:

Covered under ⇒ PANCE Blueprint Reproductive System ⇒ Complicated Pregnancy ⇒ Gestational diabetes

Also covered as part of the Women’s Health EOR topic list

2. A 60-year-old male with a history of alcohol abuse and esophageal varices is brought to the ER with lethargy, delirium, weakness, and nausea. He is normotensive and afebrile. On physical exam, he is ill-appearing with jaundice, spider angiomas, a distended abdomen, and 3+ pretibial pitting edema. Based on his history and clinical presentation, which of the following electrolyte abnormalities would you expect to see in this patient?

A. Hyponatremia
B. Hypocalcemia
C. Hypercalcemia
D. Hyperphosphatemia
E. Hypermagnesemia

Answer and topic summary

The answer is A. Hyponatremia

The patient has hypervolemic hyponatremia secondary to cirrhosis. The causes of hypervolemic hyponatremia are cirrhosis, nephrotic syndrome, and CHF. Symptoms include nausea, headache, lethargy, and seizures. It’s important to have an approach to hyponatremia since it is the most common electrolyte abnormality in the hospital.

  • First, it’s important to rule out pseudohyponatremia due to proteins, glucose, or mannitol. Also, make sure it’s not a diuretic causing hyponatremia.
  • Next, consider the volume status – are they hypervolemic, hypovolemic, or euvolemic?
  • Hypovolemic causes are more obvious (emesis, hemorrhage, etc.); however, urinary sodium can help differentiate between hypovolemia and euvolemia. If uNA < 20, then this means the renin-angiotensin-aldosterone system is on and trying to maintain pressure/volume; therefore, it is likely the patient is hypovolemic.

Treatment of hyponatremia depends on the cause. It usually involves fluid restriction and possibly (and carefully) a hypertonic solution. Remember – rapid correction of hyponatremia can lead to central pontine myelinolysis.

Smarty PANCE Content Blueprint Review:

Covered under ⇒ PANCE Blueprint Renal System ⇒ Fluid and Electrolyte Disorders ⇒ Hyponatremia

Also covered as part of the Internal Medicine EOR topic list

3. A 12-year-old male patient presents with a circular, expanding rash located where a tick had bitten him. He had recently been camping in the upper Midwest. He also complains of a headache and mild fever. Which of the following is a known cardiac complication of the patient’s most likely diagnosis?

A. Supraventricular tachycardia
B. Atrioventricular block
C. Wandering atrial pacemaker
D. Sinus bradycardia
E. None of the above

Answer and topic summary

The answer is B. Atrioventricular block

The patient most likely has Lyme disease, which is a tick-borne illness usually caused by Borrelia burgdorferi. It is more prevalent in the Northeast and upper Midwest. Erythema migrans, which is a circular, expanding rash that may look like a “bull’s eye” is seen about 20% of the time with Lyme disease. Clinical features in the early localized disease include fever, headaches, malaise, arthralgia, and lymphadenopathy. Treatment is doxycycline for non-pregnant adults and children. Amoxicillin is a second-line treatment. A well-known cardiac complication is an atrioventricular block.

Smarty PANCE Content Blueprint Review:

Covered under ⇒ PANCE Blueprint Infectious Disease ⇒ Spirochetal Disease ⇒ Lyme disease

Also covered as part of the Internal Medicine EOR and Family Medicine EOR topic list

4. Which of the following is not a manifestation of congenital rubella syndrome?

A. Deafness
B. Cataracts
C. Heart defects
D. Microcephaly
E. Fetal growth acceleration

Answer and topic summary

The answer is E. Fetal growth acceleration

Clinical features of a congenital rubella infection (CRI) include deafness, cataracts, and cardiac disease. Patients may also have CNS abnormalities, such as microcephaly. Typically, a congenital rubella infection will lead to fetal growth restriction, not fetal growth acceleration. A classic finding that you may see on exams is petechiae and purpura — some call these “blueberry muffin lesions.” In general, CRI should be considered in patients who have a suspected rubella infection during their pregnancy. Laboratory studies on the child (before the age of one) are done to help confirm the diagnosis. There is no treatment, and supportive care/surveillance is the main way to manage a congenital rubella infection.

Smarty PANCE Content Blueprint Review:

Covered under ⇒ PANCE Blueprint Infectious Disease ⇒ Viral Infectious Disease ⇒ Rubella (German Measles)

Also covered as part of the Pediatric EOR topic list

5. Which of the following is the most common cause of cardiogenic shock?

A. Right ventricle failure
B. Tachydysrhythmias
C. Myocardial infarction
D. Coronary vasospasms
E. Papillary muscle rupture

Answer and topic summary

The answer is C. Myocardial Infarction

Cardiogenic shock is a type of shock that occurs due to cardiac pump failure; essentially, it is the most severe expression of heart failure. It is defined by systemic tissue hypoperfusion secondary to poor cardiac output despite adequate intravascular volume. The most common cause of cardiogenic shock is a myocardial infarction. Other causes include atrial/ventricular tachyarrhythmias, aortic or mitral valve insufficiency, septal defects, free wall rupture, etc. Therapy options vary but may include inotropic support (e.g., dobutamine), intra-aortic balloon pump, and percutaneous ventricular assist devices.

Smarty PANCE Content Blueprint Review:

Covered under ⇒ PANCE Blueprint Cardiology ⇒ Hypotension ⇒ Cardiogenic shock

Also covered as part of the Emergency Medicine EOR topic list

6. A 20-year-old female with asthma presents to the clinic with “white stuff” in her mouth for the past few days. On physical exam, you notice a few white plaques on the buccal mucosa and tongue. The lesions are easily scraped off with a tongue depressor. Which of the following is the best initial treatment of choice for the most likely diagnosis?

A. Fluconazole
B. Amphotericin B
C. Lidocaine solution
D. Nystatin
E. Bactrim

Answer and topic summary

The answer is D. Nystatin

The patient has oropharyngeal candidiasis, which is a local infection in patients with certain risk factors (e.g., AIDs, dentures, radiation, inhaled glucocorticoids, etc.). This patient has asthma so the likely causative culprit is her inhaled glucocorticoids. The most common cause of oropharyngeal candidiasis is C. albicans. On a physical exam, you will see white plaques in the mouth (which can be scrapped). In smokers, the lesions may look more yellow/brown.

Diagnosis is usually clinical but can be confirmed by scraping the lesion. A KOH test can be performed on the scrapings — budding yeast with pseudohyphae are seen. The treatment of choice is nystatin swish and swallow or topical clotrimazole. If a patient has a severe disease or fails topical therapy, oral fluconazole can be given.

Smarty PANCE Content Blueprint Review:

Covered under ⇒ PANCE Blueprint Dermatology ⇒ Fungal Infections ⇒ Candidiasis

Also covered as part of the Internal Medicine EOR and Pediatric Rotation EOR topic list

7. A 51-year-old male presents to the clinic. His wife explains his symptoms: brief, involuntary movements of his limbs, depression, impaired thinking, and weight loss. Which of the following is the most likely diagnosis?

A. Parkinson disease
B. Multiple sclerosis
C. Alzheimer disease
D. Vascular dementia
E. Huntington’s disease

Answer and topic summary

The answer is E. Huntington’s disease

Huntington’s disease is an autosomal-dominant disorder characterized by progressive neurodegeneration. It is due to the expansion of the cytosine-adenine-guanine (CAG) trinucleotide in the HTT gene. The main clinical features include choreiform movementsdementiacognitive impairment, and psychiatric problems. Patients typically present during mid-life. Diagnosis is confirmed by genetic testing. A brain MRI may reveal caudate atrophy. Treatment is generally supportive and symptomatic. There is no cure for Huntington’s disease.

Smarty PANCE Content Blueprint Review:

Covered under ⇒ PANCE Blueprint Neurology ⇒ Movement Disorders ⇒ Huntington Disease

Also covered as part of the Internal Medicine EOR topic list

8. A 42-year-old male with a history of diabetes presents the ER with a “bad rash” on his left thigh. He said he had previously gotten a bug bite there. He is febrile (102.2F), tachycardic (110 bpm), and hypotensive (80/50 mmHg). On physical exam, you notice a 2-inch erythematous circular lesion that is warm to the touch. Which of the following is the best next step in the management of this patient?

A. Initiate IV antibiotics and fluids
B. Surgical consult for amputation
C. Outpatient dermatology referral
D. Order venous ultrasound of the leg
E. Send home on oral antibiotics

Answer and topic summary

The answer is A. Initiate IV antibiotics and fluids

This patient has a severe case of cellulitis and sepsis. The best next step is to initiate intravenous antibiotics and fluids. A broad-spectrum antibiotic (like cefepime) plus coverage for MRSA (vancomycin) would be appropriate.

The most common cause of cellulitis is Group A beta-hemolytic Streptococcus pyogenes. What differentiates cellulitis from erysipelas is that erysipelas is more well-defined. Antibiotic options for more mild cellulitis cases include Bactrimclindamycin, and Keflex.

It’s important to keep in mind that events don’t happen in a vacuum — the bug bite led to cellulitis, and the patient’s body responded to the cutaneous infection in a complex and dysregulated way (i.e. sepsis), eventually leading to hypotension and instability.

Smarty PANCE Content Blueprint Review:

Covered under ⇒ PANCE Blueprint Dermatology ⇒ Dermatologic Infectious diseases ⇒ Bacterial Infections ⇒ Cellulitis

Also covered as part of the Family Medicine EOREmergency Medicine EOR, and General Surgery EOR topic list

9. An 8-year-old male is brought into the clinic by his mother, who is concerned about his behavior in school and at home for the past 7 months. In both settings, he lacks the ability to pay attention, does not follow directions or listen, is easily distracted, is reluctant to do tasks, and constantly loses his homework and pencils. Which of the following is a common side effect of the first-line treatment for this condition?

A. Cognitive fog
B. Appetite suppression
C. Constipation
D. Somnolence
E. Xerosis

Answer and topic summary

The answer is B. Appetite suppression

The patient has attention-deficit hyperactivity disorder (ADHD). There are two main patterns: inattentive or hyperactive-impulse (some may have a combined presentation). The DSM V has criteria for diagnosing ADHD and the various subtypes, but in general, symptoms should be seen in multiple settings, last for 6 months or more, and negatively impact social/academic activities. The first-line pharmacological treatment for ADHD, in most cases, is a stimulant (e.g., methylphenidate, dextroamphetamine, etc.). The most adverse effects of stimulants include moodiness and irritability, and appetite suppression. It’s important to monitor children’s growth and weight when they are taking a stimulant for this very reason.

Smarty PANCE Content Blueprint Review:

Covered under ⇒ PANCE Blueprint Psychiatry ⇒ Neurodevelopmental disorders ⇒ Attention-Deficit/Hyperactivity Disorder

Also covered as part of the Pediatric Rotation and Psychiatry EOR topic list

10. Which of the following patients should be screened for lung cancer, according to the USPSTF?

A. 82-year-old with 34 pack-years who quit 2 years ago
B. 51-year-old with 31 pack-years who quit 10 years ago
C. 70-year-old with 14 pack-years who quit 5 years ago
D. 40-year-old with 20 pack-years who quit 2 weeks ago

Answer and topic summary

The answer is B. 51-year-old with 31 pack-years who quit 10 years ago

The U.S. Preventive Services Task Force (USPSTF) recently changed the lung cancer screening guidelines in March 2021.

An annual low-dose chest CT scan (NOT chest x-ray) should be done for those who are 50 to 80 years old with at least a 20-pack-year history and who currently smoke or have quit within the past 15 years.

It is also important to note that these guidelines do not apply to those who have another illness that already severely limits their life expectancy.

Smarty PANCE Content Blueprint Review:

Covered under ⇒ PANCE Blueprint Pulmonary ⇒ Pulmonary Neoplasms ⇒ Lung cancer

Also covered as part of the Family Medicine EOR and Emergency Medicine EOR topic list

This podcast is available on every device!

You can download and listen to past FREE episodes here, on iTunes, Spotify, Google Podcasts, Stitcher, Amazon Music, and all podcasting apps.

Download Interactive Content Blueprint Checklists for the PANCE, PANRE, EOR, and PANRE-LA

Interactive Content Blueprints for the PANCE PANRE and PANRE-LA

Follow this link to download your FREE copy of the PANCE/PANRE/EOR Content Blueprint Checklists.

Print it up and start crossing out the topics you understand, marking the ones you don’t, and making notes of key terms you should remember. The PDF version is interactive and linked directly to the individual lessons on Smarty PANCE.

The post Podcast Episode 104: Ten PANCE, PANRE, and Rotation Review Questions appeared first on The Audio PANCE and PANRE.

]]>
Listen to Podcast Episode 104: Ten PANCE, PANRE, and Rotation Review Questions If you can’t see the audio player, click here to listen to the full episode. Welcome to episode 104 of the Audio PANCE and PANRE Physician Assistant/Associate Board Review P... If you can’t see the audio player, click here to listen to the full episode.
Welcome to episode 104 of the Audio PANCE and PANRE Physician Assistant/Associate Board Review Podcast.
Join me today as we cover ten board review questions for your PANCE, PANRE, EOR, and EOC exams.
Links from today’s episode:

* Sign up for the Entire Blueprint Email Series
* Follow Smarty PANCE and The Daily PANCE Blueprint on Instagram
* Follow Smarty PANCE and The Daily PANCE Blueprint on Facebook
* Join the Smarty PANCE Member’s Community, then sign up for a study group to get updates about upcoming webinars.

I hope you enjoy this free audio component of the examination portion of this site. Smarty PANCE includes over 2,000 interactive board review questions, along with flashcards, ReelDx cases, integrated Picmonics, and lessons covering every blueprint topic available to all Smarty PANCE members.

* You can download and listen to past FREE episodes here, on iTunes, Spotify, Google Podcasts, Stitcher, Amazon Music, and all podcasting apps
* You can listen to all the latest episodes, take interactive quizzes, and download more resources on each episode page.

Smarty PANCE is not sponsored or endorsed by, or affiliated with, the National Commission on Certification of Physician Assistants.
Interactive exam to complement today’s podcast
When is screening for gestational diabetes done?
A. 16 weeks
B. 22 weeks
C. 24 weeks
D. 32 weeks
E. 34 weeks
2. A 60-year-old male with a history of alcohol abuse and esophageal varices is brought to the ER with lethargy, delirium, weakness, and nausea. He is normotensive and afebrile. On physical exam, he is ill-appearing with jaundice, spider angiomas, a distended abdomen, and 3+ pretibial pitting edema. Based on his history and clinical presentation, which of the following electrolyte abnormalities would you expect to see in this patient?
A. Hyponatremia
B. Hypocalcemia
C. Hypercalcemia
D. Hyperphosphatemia
E. Hypermagnesemia
3. A 12-year-old male patient presents with a circular, expanding rash located where a tick had bitten him. He had recently been camping in the upper Midwest. He also complains of a headache and mild fever. Which of the following is a known cardiac complication of the patient’s most likely diagnosis?
A.]]>
The Physician Assistant Life | Smarty PANCE full 24:25 568
Podcast Episode 103: Ten PANRE & PANRE-LA Intervention Complex Practice Questions http://podcast.thepalife.com/podcast-episode-103/ Wed, 08 Mar 2023 12:00:42 +0000 https://podcast.thepalife.com/?p=562 Listen to Podcast Episode 103: Ten PANRE & PANRE-LA Intervention Complex Practice Question If you can’t see the audio player, click here to listen to the full episode. Welcome to episode 103 of the Audio PANCE and PANRE Physician Assistant/Associate Board Review Podcast. Join me today as we cover ten NCCPA-style board review questions for […]

The post Podcast Episode 103: Ten PANRE & PANRE-LA Intervention Complex Practice Questions appeared first on The Audio PANCE and PANRE.

]]>
Listen to Podcast Episode 103: Ten PANRE & PANRE-LA Intervention Complex Practice Question

If you can’t see the audio player, click here to listen to the full episode.

Podcast Episode 103 Ten PANRE & PANRE-LA Intervention Complex Practice QuestionsWelcome to episode 103 of the Audio PANCE and PANRE Physician Assistant/Associate Board Review Podcast.

Join me today as we cover ten NCCPA-style board review questions for your PANRE and PANRE-LA exams.

Special from today’s episode:

I hope you enjoy this free audio component of the examination portion of this site. Smarty PANCE includes over 2,000 interactive board review questions, along with flashcards, ReelDx cases, integrated Picmonics, and lessons covering every blueprint topic available to all Smarty PANCE members.

Here is an interactive exam to complement today’s podcast

1. A 65-year-old man presents to your office with complaints of constipation for the past six months. He says that he has difficulty passing stools, which are hard and dry. He also reports occasional abdominal pain and bloating. He denies any weight loss, blood in stools, fever, or night sweats. His medical history is significant for hypertension and type 2 diabetes mellitus. His medications include metformin, lisinopril, and aspirin. He does not smoke or drink alcohol. On physical examination, his vital signs are normal. His abdomen is soft and nontender, with normal bowel sounds. There are no masses or organomegaly palpable. Which of the following is the most appropriate next step in evaluating this patient?

A) Colonoscopy
B) Barium enema
C) Thyroid function tests
D) Stool osmolarity
E) Dietary modification

Answer and topic summary

The correct answer is A) Colonoscopy

Colonoscopy is a procedure that involves inserting a flexible tube with a camera into the colon to visualize the mucosa and detect any abnormalities such as polyps, tumors, inflammation, or bleeding. It is indicated for patients with chronic constipation who are older than 50 years or have any red flag features for colorectal malignancy, such as weight loss, blood in stools, anemia, or a family history of colon cancer. This patient meets the age criterion and should undergo colonoscopy to rule out any serious causes of his constipation.

Answer explanations:

  • Barium enema is an imaging test that involves injecting a contrast agent (barium sulfate) into the rectum and taking X-rays of the colon. It can show structural abnormalities such as diverticula, strictures, masses, or volvulus. However, it is less sensitive and specific than colonoscopy for detecting colorectal malignancy. Therefore, it is not the preferred test for this patient.
  • Thyroid function tests are blood tests that measure the levels of thyroid hormones (T3, T4) and thyroid-stimulating hormone (TSH). They can help diagnose thyroid disorders such as hypothyroidism or hyperthyroidism. Hypothyroidism can cause constipation due to decreased gastrointestinal motility. However, this patient has no other signs or symptoms of hypothyroidism such as fatigue, cold intolerance, dry skin, hair loss, or bradycardia. Therefore, thyroid function tests are not necessary for this patient.
  • Stool osmolarity is a test that measures the concentration of solutes in stool water. It can help differentiate between osmotic diarrhea (high stool osmolarity) and secretory diarrhea (low stool osmolarity). However, this test is not useful for evaluating constipation which is defined by infrequent or difficult defecation.
  • Dietary modification is a nonpharmacological measure that involves increasing fiber and fluid intake to improve stool consistency and frequency. It may be effective for patients with primary constipation which has no identifiable organic cause. However, this patient has chronic constipation which requires further evaluation before initiating treatment.

(Review PANRE Blueprint Topic: Constipation)

2. A 60-year-old male presents to the emergency department with chest pain and shortness of breath. ECG reveals ST segment elevation in the anterior leads. Which of the following laboratory tests is the most specific for the diagnosis of acute myocardial infarction (AMI)?

A) Troponin
B) Creatinine kinase-MB (CK-MB)
C) Myoglobin
D) C-reactive protein (CRP)
E) Brain natriuretic peptide (BNP)

Answer and topic summary

The correct answer is A) Troponin

Acute myocardial infarction (AMI) is a serious medical emergency that requires prompt diagnosis and treatment. Cardiac biomarkers such as troponin, creatinine kinase-MB (CK-MB), and myoglobin are commonly used to diagnose AMI. Among these biomarkers, troponin is the most specific for the diagnosis of AMI. Troponin is a protein found in cardiac muscle cells, and its release into the bloodstream is a specific marker of myocardial injury. Elevated troponin levels are typically seen within 3-4 hours after the onset of symptoms and can remain elevated for up to 10-14 days after an AMI.

Incorrect answer explanations:

  • Creatinine kinase (CK) is an enzyme found in various tissues, including skeletal and cardiac muscle. CK-MB is a specific isoform of CK found predominantly in cardiac muscle cells. Elevated levels of CK-MB can be seen in the early stages of AMI, but CK-MB is less specific for the diagnosis of AMI compared to troponin.
  • Myoglobin is a protein found in skeletal and cardiac muscle cells. Elevated myoglobin levels can be seen within 1-3 hours after the onset of symptoms, but myoglobin is less specific for the diagnosis of AMI compared to troponin. Elevated myoglobin levels can also be seen in other conditions that cause muscle injury, such as rhabdomyolysis.
  • C-reactive protein (CRP) is an acute-phase protein that is elevated in response to tissue injury, inflammation, and infection. While elevated CRP levels can be seen in patients with AMI, CRP is not specific for the diagnosis of AMI and cannot be used as a diagnostic tool on its own.
  • Brain natriuretic peptide (BNP) is a hormone released by the heart in response to increased pressure and volume in the cardiac chambers. Elevated BNP levels can be seen in patients with heart failure and other cardiac conditions, but BNP is not specific for the diagnosis of AMI.

(Review PANRE Blueprint Topic: Acute myocardial infarction: ST-segment elevation myocardial infarction (STEMI))

3. A 65-year-old woman with a history of atrial fibrillation, hypertension, and obesity presents to the emergency department with acute onset of severe left leg pain and swelling. She has been taking warfarin for anticoagulation but admits to missing some doses in the past week. Her blood pressure is 180/100 mmHg, pulse is 110 beats/min and irregular, and respiratory rate is 20 breaths/min. Her physical examination reveals a warm, tender, erythematous left lower extremity with prominent superficial veins and a positive Homan’s sign. Her international normalized ratio (INR) is 1.5. What is the most appropriate next step in the management of this patient?

A) Start heparin infusion and adjust warfarin dose
B) Order duplex ultrasonography of the lower extremities
C) Administer tissue plasminogen activator (tPA)
D) Perform venography of the left leg
E) Apply compression stockings and elevate the leg

Answer and topic summary

The correct answer is B) Order duplex ultrasonography of the lower extremities

Duplex ultrasonography is a non-invasive test that uses sound waves to visualize the blood flow in the veins of the legs. It can detect thrombi by measuring the diameter, compressibility, and flow characteristics of the veins. It has high sensitivity and specificity for diagnosing DVT. It is considered the first-line diagnostic test for patients with suspected DVT. If negative, it can be repeated in one week or combined with D-dimer testing to rule out DVT.

Incorrect answer explanations:

  • Heparin infusion and warfarin dose adjustment are indicated for patients with confirmed deep vein thrombosis (DVT) who have subtherapeutic INR levels. However, this patient has not yet been diagnosed with DVT and needs further diagnostic testing before initiating anticoagulation therapy.
  • tPA is a thrombolytic agent that dissolves blood clots by activating plasminogen into plasmin. Plasmin degrades fibrin, which forms the meshwork of clots. tPA may be used for patients with massive or life-threatening DVT who have contraindications to anticoagulation or who have failed anticoagulation therapy. However, it carries a high risk of bleeding complications and should not be used without confirming DVT diagnosis first.
  • Venography is an invasive procedure that involves injecting contrast dye into a vein and taking X-ray images to visualize any obstruction or narrowing in the venous system. It was once considered the gold standard for diagnosing DVT but has been largely replaced by duplex ultrasonography due to its higher cost, invasiveness, discomfort, and risk of allergic reactions or nephrotoxicity from contrast dye.
  • Compression stockings are elastic garments that apply graduated pressure on the legs to improve venous return and prevent edema formation. They may be used as an adjunctive treatment for patients with confirmed DVT, along with anticoagulation therapy. Elevation of the leg can also reduce swelling and pain by decreasing hydrostatic pressure in the veins. However, these interventions do not address the underlying cause of DVT or prevent clot propagation or embolization. They should not be used as the sole therapy for suspected or confirmed DVT without diagnostic testing or anticoagulation therapy.

(Review PANRE Blueprint Topic: Deep venous thrombosis)

4. A 7-year-old child presents with honey-colored crusting lesions on the face and extremities. The lesions started as small red papules that quickly progressed to vesicles and then ruptured, leaving behind a honey-colored crust. The child has no fever and is otherwise healthy. What is the most appropriate initial treatment for this condition?

A) Topical corticosteroids
B) Oral antihistamines
C) Oral doxycycline
D) Topical mupirocin
E) Oral acyclovir

Answer and topic summary

The correct answer is D) Topical mupirocin

The presentation of honey-colored crusting lesions on the face and extremities in a child is consistent with impetigo, a bacterial skin infection caused by Staphylococcus aureus or Streptococcus pyogenes. The most appropriate initial treatment for impetigo is topical antibiotics such as mupirocin. This is because impetigo is a localized skin infection, and topical antibiotics can effectively treat it without the need for systemic antibiotics. The topical antibiotic is applied to the affected area 2-3 times per day for 5-7 days.

Incorrect answer explanations:

  • Topical corticosteroids are not appropriate for impetigo as they can exacerbate the infection by suppressing the immune response and promoting bacterial growth.
  • Oral antihistamines are used to treat allergic reactions and are not effective in treating bacterial skin infections such as impetigo.
  • Oral doxycycline can be used to treat impetigo, but it is not the initial treatment of choice. Oral antibiotics are reserved for cases of extensive or severe impetigo or for patients who cannot tolerate topical antibiotics.
  • Oral acyclovir is an antiviral medication used to treat viral skin infections such as herpes simplex virus (HSV). It is not effective in treating bacterial skin infections such as impetigo.

(Review PANRE Blueprint Topic: Impetigo)

5. A 72-year-old woman with a history of hypertension, diabetes mellitus, and atrial fibrillation presents to the emergency department with sudden onset of left-sided weakness and slurred speech. She was last seen normal 2 hours ago by her daughter. Her vital signs are: blood pressure 180/100 mmHg, pulse 110 beats/min irregularly irregular, respiratory rate 18 breaths/min, temperature 36.5°C (97.7°F), and oxygen saturation 98% on room air. On physical examination, she has left facial droop, left hemiparesis (4/5 strength), and dysarthria. Her NIH Stroke Scale score is 8. A non-contrast head CT scan shows no evidence of hemorrhage. What is the most appropriate next step in management?

A) Administer intravenous alteplase
B) Administer intravenous heparin
C) Perform carotid endarterectomy
D) Perform mechanical thrombectomy
E) Start oral aspirin

Answer and topic summary

The correct answer is A. Administer intravenous alteplase

The patient has a clinical diagnosis of acute ischemic stroke, which is caused by occlusion of a cerebral artery by a thrombus or embolus. The most important factor in determining the treatment of acute ischemic stroke is the time from symptom onset to presentation. If the patient presents within 4.5 hours of symptom onset and has no contraindications, intravenous alteplase (a tissue plasminogen activator or tPA) should be administered as soon as possible to dissolve the clot and restore blood flow to the ischemic brain tissue. Intravenous alteplase has been shown to improve functional outcomes and reduce disability after acute ischemic stroke. The patient meets the criteria for intravenous alteplase because she presented within 4.5 hours of symptom onset, has no evidence of hemorrhage on head CT scan, and has no other contraindications such as recent surgery, active bleeding, severe hypertension (>185/110 mmHg), or use of anticoagulants. The other answer choices are incorrect because they are not indicated or effective in this scenario.

Incorrect answer explanations:

  • Intravenous heparin administration is not recommended for acute ischemic stroke because it does not improve outcomes and may increase the risk of bleeding complications.
  • Carotid endarterectomy is a surgical procedure that removes plaque from the carotid artery to prevent recurrent strokes in patients with significant carotid stenosis (>70%) who are asymptomatic or have had a transient ischemic attack (TIA) or minor stroke within the past 6 months. It is not indicated for patients with acute ischemic stroke who have not been stabilized medically.
  • Mechanical thrombectomy is an endovascular procedure that uses a catheter-based device to remove large vessel occlusions in patients with acute ischemic stroke who have a large penumbra (area of potentially salvageable brain tissue). It can be performed within 24 hours of symptom onset if intravenous alteplase is contraindicated or ineffective. However, it should not delay or replace intravenous alteplase if eligible.
  • Oral aspirin is recommended for secondary prevention of ischemic stroke after initial treatment with intravenous alteplase or mechanical thrombectomy. It can also be used as initial treatment for patients who present more than 24 hours after symptom onset or who have contraindications to intravenous alteplase. However, it should not be given within 24 hours of receiving intravenous alteplase because it may increase the risk of bleeding complications.

(Review PANRE Blueprint Topic: Stroke)

[dt_sc_button type=”type2″ link=”https://smartypance.com/lessons/vascular-disorders/stroke-reeldx266/” size=”small” target=”_blank” timeline_button=”no”]View blueprint lesson[/dt_sc_button]

6. A 65-year-old man with a history of hypertension and peptic ulcer disease presents to your clinic for a routine follow-up. He reports feeling well and has no complaints. His medications include lisinopril and omeprazole. His vital signs are normal. A complete blood count (CBC) shows:

  • Hemoglobin: 10 g/dL (normal: 13-17 g/dL)
  • Hematocrit: 30% (normal: 40-50%)
  • Mean corpuscular volume (MCV): 70 fL (normal: 80-100 fL)
  • Red cell distribution width (RDW): 18% (normal: 11-15%)
  • White blood cell count: 6 x 10^9/L (normal: 4-11 x 10^9/L)
  • Platelet count: 250 x 10^9/L (normal: 150-450 x 10^9/L)

What is the most likely cause of this patient’s anemia?

A) Chronic kidney disease
B) Folate deficiency
C) Gastrointestinal bleeding
D) Thalassemia trait
E) Vitamin B12 deficiency

Answer and topic summary

The answer is C. Gastrointestinal bleeding

The patient has a microcytic anemia, which is characterized by a low MCV (<80 fL). The most common cause of microcytic anemia is iron deficiency. Iron deficiency can result from inadequate dietary intake, malabsorption, increased demand, or chronic blood loss. In this patient, the most likely source of chronic blood loss is gastrointestinal bleeding due to his history of peptic ulcer disease and the use of omeprazole. Omeprazole can mask the symptoms of gastrointestinal bleeding by reducing acid secretion and healing ulcers but does not prevent recurrence or complications. The patient may also have occult blood loss that is not visible in the stool. The RDW is elevated (>15%), indicating increased variation in red blood cell size due to iron deficiency. To confirm iron deficiency, iron studies such as serum ferritin, serum iron, total iron-binding capacity (TIBC), and transferrin saturation should be ordered. The patient should also undergo endoscopy to evaluate for the source and severity of gastrointestinal bleeding and rule out malignancy. The other answer choices are incorrect because they are not causes of microcytic anemia but rather causes of normocytic or macrocytic anemia.

Incorrect answer explanations:

  • Chronic kidney disease can cause normocytic anemia due to reduced production of erythropoietin by the kidneys. The MCV would be normal (80-100 fL).
  • Folate deficiency can cause macrocytic anemia due to impaired DNA synthesis in red blood cell precursors. The MCV would be high (>100 fL).
  • Thalassemia trait can cause microcytic anemia due to reduced synthesis of alpha or beta globin chains that form hemoglobin. However, thalassemia trait usually has a normal or low RDW (<15%) because red blood cells are uniformly small. Thalassemia trait also has a genetic basis and is more common in people of Mediterranean, African, or Southeast Asian descent.
  • Vitamin B12 deficiency can also cause macrocytic anemia due to impaired DNA synthesis in red blood cell precursors as well as neurological symptoms such as peripheral neuropathy, ataxia, dementia, or psychosis. The MCV would be high (>100 fL).

(Review PANRE Blueprint Topic: Iron Deficiency Anemia)

7. A 42-year-old woman with a history of GERD presents to your clinic for follow-up. She has been taking omeprazole 20 mg daily for the past 6 months and reports significant improvement in her heartburn and regurgitation symptoms. She has also made lifestyle modifications such as avoiding spicy and fatty foods, quitting smoking, and elevating the head of her bed. She asks you if she can stop taking omeprazole or reduce the dose. What is the most appropriate next step in management?

A) Continue omeprazole 20 mg daily indefinitely
B) Discontinue omeprazole and monitor symptoms
C) Switch to famotidine 20 mg twice daily
D) Taper omeprazole to every other day for 4 weeks
E) Perform an upper endoscopy

Answer and topic summary

The correct answer is D) Taper omeprazole to every other day for 4 weeks

Explanation:

The patient has a history of GERD that has responded well to PPI therapy and lifestyle modifications. The goal of treatment is to achieve symptom relief and prevent complications such as esophagitis, stricture, Barrett’s esophagus, or adenocarcinoma. PPIs are more effective than H2 blockers or antacids for healing erosive esophagitis and maintaining remission. However, long-term use of PPIs may be associated with adverse effects such as increased risk of fractures, infections, hypomagnesemia, vitamin B12 deficiency, and chronic kidney disease. Therefore, it is reasonable to attempt a step-down approach after achieving symptom control with PPIs for at least 8 weeks. This involves tapering the dose of PPI gradually over 2-4 weeks to avoid rebound acid hypersecretion. If symptoms recur after discontinuation of PPIs, then switching to an H2 blocker or restarting PPI maintenance therapy with the lowest effective dose may be indicated.

Incorrect answer choices:

  • Continuing omeprazole 20 mg daily may expose the patient to unnecessary risks of long-term PPI use without attempting a trial of dose reduction or discontinuation.
  • Discontinuing omeprazole and monitoring symptoms may cause rebound acid hypersecretion and recurrence of GERD symptoms due to abrupt withdrawal of PPI therapy.
  • Immediately switching to an H2 blocker (such as famotidine) may not provide adequate symptom control for GERD, as H2 blockers are less potent than PPIs in suppressing gastric acid secretion. Slowly taper off the PPI first over 2-4 weeks (the higher the dose, the longer the taper). If symptoms return, it would be appropriate to start again with an H2 blocker.  If long-term treatment is needed, H2 blockers allow better absorption of nutrients than PPIs and so potentially have fewer long-term adverse effects. If symptoms are still difficult to control, consider adding the PPI back at the lowest effective dose.
  • Perform an upper endoscopy: This option is not indicated for patients with uncomplicated GERD who have responded well to medical therapy. Endoscopy is reserved for patients who have alarm features such as dysphagia, odynophagia, weight loss, anemia,

(Review PANRE Blueprint Topic: GERD)

8. Which of the following is an appropriate step-down therapy for a patient with well-controlled asthma on medium-dose inhaled corticosteroids (ICS) and long-acting beta agonists (LABA)?

A) Discontinue LABA and continue medium-dose ICS
B) Discontinue ICS and continue LABA
C) Reduce ICS dose by 50% and continue LABA
D) Reduce both ICS and LABA doses by 50%
E) Switch to low-dose ICS/formoterol as needed

Answer and topic summary

The answer is E. Switch to low-dose ICS/formoterol as needed

This patient has well-controlled asthma on medium-dose ICS and LABA, which corresponds to step 4 of the asthma treatment algorithm. Step-down therapy can be considered for patients with at least 3 months of continuous good control of asthma. The goal of step-down therapy is to reduce medication use to the lowest effective dose while maintaining asthma control. According to the Global Initiative for Asthma (GINA), one option for stepping down from step 4 is to switch to low-dose ICS/formoterol as needed. This regimen involves using a combination inhaler containing low-dose ICS (budesonide) and formoterol (a fast-acting LABA) both as maintenance therapy and as reliever therapy instead of a short-acting beta agonist (SABA). This strategy has been shown to reduce exacerbations, improve symptom control, and decrease steroid exposure compared with conventional maintenance therapy with higher doses of ICS/LABA plus SABA as needed.

(Review PANRE Blueprint Topic: Asthma)

9. A 35-year-old woman presents to her primary care provider with complaints of chronic worry and nervousness for the past 8 months. She says she worries about everything, such as her health, her family, her work, and her finances. She has difficulty sleeping, concentrating, and relaxing. She also experiences muscle tension, headaches, and palpitations. She denies any history of trauma, substance abuse, or other psychiatric disorders. She has no medical problems and takes no medications. Her vital signs are normal and her physical examination is unremarkable. Which of the following is the most likely diagnosis?

A) Panic disorder
B) Obsessive-compulsive disorder
C) Post-traumatic stress disorder
D) Generalized anxiety disorder
E) Adjustment disorder

Answer and topic summary

The correct answer is D) Generalized anxiety disorder

Generalized anxiety disorder (GAD) is characterized by persistent, excessive, and unrealistic worry about everyday things that lasts for at least 6 months. The worry causes significant distress or impairment in social, occupational, or other areas of functioning. The anxiety is not attributable to any specific triggers or stressors. The diagnosis of GAD requires at least 3 of the following symptoms: restlessness, fatigue, difficulty concentrating, irritability, muscle tension, and sleep disturbance.

  • Panic disorder is characterized by recurrent unexpected panic attacks that cause fear of having another attack or avoidance of situations that might trigger an attack. Panic attacks are sudden episodes of intense fear or discomfort that peak within minutes and are accompanied by at least 4 physical or cognitive symptoms such as palpitations, sweating, trembling, shortness of breath, chest pain, nausea, dizziness, derealization/depersonalization, fear of losing control or dying. The patient does not report having panic attacks.
  • Obsessive-compulsive disorder (OCD) is characterized by recurrent obsessions (intrusive thoughts or images that cause anxiety) and/or compulsions (repetitive behaviors or mental acts that aim to reduce anxiety). The patient does not report having obsessions or compulsions.
  • Post-traumatic stress disorder (PTSD) is characterized by exposure to a traumatic event that involved actual or threatened death, serious injury, or sexual violence, and subsequent re-experiencing, avoidance, negative alterations in cognition and mood, and increased arousal related to the event. The patient denies any history of trauma.
  • Adjustment disorder is characterized by emotional or behavioral symptoms that develop within 3 months of an identifiable psychosocial stressor and cause significant impairment in social, occupational, or other areas of functioning. The symptoms usually resolve within 6 months after the termination of the stressor unless it has chronic consequences. The patient’s anxiety is not related to any specific stressors and has lasted longer than 6 months.

(Review PANRE Blueprint Topic: Generalized Anxiety Disorder)

10. A 65-year-old man with a history of type 2 diabetes mellitus presents to the emergency department with altered mental status, polyuria, and polydipsia. He has been feeling unwell for the past week with a urinary tract infection that he has been self-treating with cranberry juice. His vital signs are: blood pressure 180/100 mmHg, heart rate 110 beats per minute, respiratory rate 24 breaths per minute, temperature 37.8°C (100°F), and oxygen saturation 95% on room air. His physical examination reveals dry mucous membranes, poor skin turgor, and decreased level of consciousness. His laboratory tests show:

  • Serum glucose: 900 mg/dL
  • Serum sodium: 150 mEq/L
  • Serum potassium: 4.0 mEq/L
  • Serum bicarbonate: 18 mEq/L
  • Serum osmolality: 350 mOsm/kg
  • Urine ketones: negative

Which of the following is the most appropriate initial treatment?

A) Intravenous insulin infusion
B) Intravenous normal saline infusion
C) Intravenous sodium bicarbonate infusion
D) Intravenous potassium chloride infusion
E) Subcutaneous insulin glargine injection

Answer and topic summary

The correct answer is B) Intravenous normal saline infusion

This patient has hyperosmolar hyperglycemic syndrome (HHS), which is a complication of diabetes mellitus characterized by severe hyperglycemia (>600 mg/dL), hyperosmolality (>320 mOsm/kg), and dehydration in the absence of ketoacidosis. HHS is often triggered by an acute stressor such as infection, medication noncompliance, or excessive carbohydrate intake. The mainstay of treatment for HHS is fluid replacement with isotonic saline to correct dehydration, lower serum glucose and osmolality, and improve renal function. Insulin therapy can be initiated after adequate fluid resuscitation, usually at a low dose (0.05-0.1 units/kg/hour). Electrolyte abnormalities such as hypokalemia or acidosis should be corrected as needed.

Incorrect answer explanations:

  • Insulin infusion is not the first-line treatment for HHS, as it can worsen dehydration, hypokalemia, and cerebral edema if given before adequate fluid replacement. Insulin should be started after fluid resuscitation at a low dose to avoid rapid drops in serum glucose and osmolality.
  • Sodium bicarbonate infusion is not indicated for HHS unless there is severe acidosis (pH <7.0). The patient’s serum bicarbonate level is mildly low (18 mEq/L), but not enough to warrant bicarbonate therapy. Moreover, sodium bicarbonate can increase serum osmolality and worsen cerebral edema.
  • Intravenous potassium chloride infusion is unnecessary for HHS unless there is hypokalemia (<3.5 mEq/L). The patient’s serum potassium level is normal (4.0 mEq/L), so potassium supplementation is not required. However, potassium levels should be monitored closely during fluid and insulin therapy, as they may drop rapidly due to intracellular shifts.
  • Insulin glargine injection is a long-acting insulin that provides basal coverage for up to 24 hours. It is not suitable for treating acute hyperglycemia in HHS, as it has a slow onset of action and cannot be titrated easily according to blood glucose levels. Moreover, subcutaneous insulin administration may be unreliable in patients with poor perfusion due to dehydration.

This podcast is available on every device!

You can download and listen to past FREE episodes here, on iTunes, Spotify, Google Podcasts, Stitcher, Amazon Music, and all podcasting apps.

Download Interactive Content Blueprint Checklists for the PANCE, PANRE, EOR, and PANRE-LA

PANRE and PANRE LA Interactive Content Blueprints

Follow this link to download your FREE copy of the PANCE/PANRE/EOR Content Blueprint Checklists.

Print it up and start crossing out the topics you understand, marking the ones you don’t, and making notes of key terms you should remember. The PDF version is interactive and linked directly to the individual lessons on Smarty PANCE.

The post Podcast Episode 103: Ten PANRE & PANRE-LA Intervention Complex Practice Questions appeared first on The Audio PANCE and PANRE.

]]>
Listen to Podcast Episode 103: Ten PANRE & PANRE-LA Intervention Complex Practice Question If you can’t see the audio player, click here to listen to the full episode. Welcome to episode 103 of the Audio PANCE and PANRE Physician Assistant/Associate Bo... If you can’t see the audio player, click here to listen to the full episode.
Welcome to episode 103 of the Audio PANCE and PANRE Physician Assistant/Associate Board Review Podcast.
Join me today as we cover ten NCCPA-style board review questions for your PANRE and PANRE-LA exams.
Special from today’s episode:

* Take the new PANRE & PANRE-LA (Intervention Complex) Practice Exam: Covers all the topics tested within the new PANRE (Intervention Complex) performance expectation with links to Smarty PANCE lessons.
* PANRE & PANRE-LA Blueprint 8-Week Schedule and Study Planner
* Read The New 2023 PANRE and PANRE-LA: Everything you Need to Know
* Members can try out the newly updated PANRE-LA Smart Search Tool (you must log in to access the search bar)
* Sign up for the Entire Blueprint Email Series to get daily questions for the next 478 days!
* Follow Smarty PANCE and The Daily PANCE Blueprint on Instagram
* Follow Smarty PANCE and The Daily PANCE Blueprint on Facebook
* Join the Smarty PANCE Member’s Community, then sign up for a study group to get updates about upcoming webinars

I hope you enjoy this free audio component of the examination portion of this site. Smarty PANCE includes over 2,000 interactive board review questions, along with flashcards, ReelDx cases, integrated Picmonics, and lessons covering every blueprint topic available to all Smarty PANCE members.

* You can download and listen to past FREE episodes here, on iTunes, Spotify, Google Podcasts, Stitcher, and most podcasting apps.
* You can listen to all the latest episodes, take interactive quizzes, and download more resources on each episode page.

Here is an interactive exam to complement today’s podcast
1. A 65-year-old man presents to your office with complaints of constipation for the past six months. He says that he has difficulty passing stools, which are hard and dry. He also reports occasional abdominal pain and bloating. He denies any weight loss, blood in stools, fever, or night sweats. His medical history is significant for hypertension and type 2 diabetes mellitus.]]>
The Physician Assistant Life | Smarty PANCE full 34:55 562
Podcast Episode 102: Ten PANCE, PANRE, and Rotation Review Questions http://podcast.thepalife.com/podcast-episode-102-ten-pance-panre-and-rotation-review-questions/ Thu, 09 Feb 2023 23:40:51 +0000 https://podcast.thepalife.com/?p=556 Listen to Podcast Episode 102: Ten PANCE, PANRE, and Rotation Review Questions If you can’t see the audio player, click here to listen to the full episode. Welcome to episode 102 of the Audio PANCE and PANRE Physician Assistant/Associate Board Review Podcast. Join me today as we cover ten board review questions for your PANCE, […]

The post Podcast Episode 102: Ten PANCE, PANRE, and Rotation Review Questions appeared first on The Audio PANCE and PANRE.

]]>
Listen to Podcast Episode 102: Ten PANCE, PANRE, and Rotation Review Questions

If you can’t see the audio player, click here to listen to the full episode.

Podcast Episode 102: The Audio PANCE and PANREWelcome to episode 102 of the Audio PANCE and PANRE Physician Assistant/Associate Board Review Podcast.

Join me today as we cover ten board review questions for your PANCE, PANRE, EOR™, and EOC™ exams.

Special from today’s episode:

I hope you enjoy this free audio component of the examination portion of this site. Smarty PANCE includes over 2,000 interactive board review questions, along with flashcards, ReelDx cases, integrated Picmonics, and lessons covering every blueprint topic available to all Smarty PANCE members.

Interactive exam to complement today’s podcast

1. Which of the following is NOT true about a non-ST elevation myocardial infarction?

A. Non-enteric-coated, chewable aspirin 325 mg should be given
B. Troponins are elevated
C. It happens due to a partially occluded epicardial coronary artery
D. Patients need a 12-lead EKG
E. You will always see ST depressions

Answer and topic summary

The answer is E. You will always see ST depressions

A non-ST elevation myocardial infarction (NSTEMI) is defined by the absence of persistent ST-elevation with elevated cardiac biomarkers (e.g., troponin I or T, CKMB, etc). It happens due to a partially occluded epicardial coronary artery (leading to subendocardial ischemia). NSTEMI typically presents as pressure-type chest pain.

Patients with a suspected NSTEMI should receive a 12-lead EKG within 10 minutes of arrival. ST depression, transient ST-elevation, and/or T-wave inversions may be seen on EKG, but they are NOT required for the diagnosis of NSTEMI. The most important medication to give is non-enteric-coated chewable aspirin 325 mg. Other meds include sublingual nitroglycerin, oxygen as needed, beta-blockers (assuming no C/I), high-intensity statin, ACE inhibitors (if CKD, DM, or EF < 40%), P2Y12 inhibitor (e.g., clopidogrel), anticoagulation, and possibly PCI with stenting or CABG.

Smarty PANCE Content Blueprint Review:

Covered under ⇒ PANCE Blueprint Cardiology ⇒ Coronary Heart Disease ⇒ Acute myocardial infarction ⇒ Non-ST-Segment Elevation MI (NSTEMI)

Also covered as part of the Family Medicine EORInternal Medicine EOREmergency Medicine EOR topic list

2. A 22-year-old G1P0 female at 28 weeks gestation with a history of diabetes presents to the clinic with fever, chills, and dysuria. Vitals show tachycardia (115 bpm), tachypnea (22 bpm), and hypotension (90/58 mmHg). Physical exam reveals suprapubic tenderness. Labs reveal leukocytosis, hyponatremia, and hyperglycemia. Urine dipstick is positive for nitrites, blood, glucose, and ketones. Which of the following is the next best step?

A. Send home with oral antibiotics and analgesics
B. Reassurance and oral rehydration solution
C. Admit to hospital for antibiotics, fluids, & insulin
D. Order outpatient CT scan of the abdomen
E. Refer to an outpatient nephrologist for workup

Answer and topic summary

The answer is C. Admit to hospital for antibiotics, fluids, & insulin

The patient most likely has pyelonephritis, which is an infection of the upper urinary tract and kidneys. It is one of the most common causes of sepsis in pregnancy. The pyelonephritis is also precipitating diabetic ketoacidosis in this diabetic patient.

Clinical features of pyelonephritis include fever, flank pain, N/V, and possible CVA tenderness. Pregnant patients are at high risk for obstetric and medical complications from the infection. It is recommended that pregnant women with acute pyelonephritis should be admitted for IV antibiotics (typically at least until the woman is febrile for 1-2 days and symptomatically improved). Antibiotic options include cefepime, piperacillin-tazobactam, and meropenem. This patient will also need insulin and fluids.

Smarty PANCE Content Blueprint Review:

Covered under ⇒ PANCE Blueprint Genitourinary ⇒ Infectious Disorders ⇒ Pyelonephritis

Also covered as part of the Family Medicine EORInternal Medicine EOR, and Emergency Medicine EOR topic list

3. Which of the following is the most common benign neoplasm of the liver?

A. Hepatocellular carcinoma
B. Hepatic hemangioma
C. Liver angiosarcoma
D. Hepatocellular adenoma
E. Hepatoblastoma

Answer and topic summary

The answer is B. Hepatic hemangioma

Hepatic hemangiomas are the most common benign liver lesions. The typical patient is a 30 to a 50-year-old woman. Exposure to estrogen may increase the size of hepatic hemangiomas. Patients are typically asymptomatic; however, if they do have symptoms they may have RUQ abdominal pain. An ultrasound will show a homogeneous, hyperechoic mass. If lesions are < 5 cm, you usually don’t need to do anything. If > 5 cm, monitoring is needed Q6-12 months via MRI.

Smarty PANCE Content Blueprint Review:

Covered under ⇒ PANCE Blueprint GI and Nutrition ⇒ Gastrointestinal System Neoplasms ⇒ Liver neoplasms

4. A 12-year-old male presents with a sore throat, fever, dysphagia, and a muffled voice. On physical exam, the uvula is deviated. Which of the following is the most likely diagnosis?

A. Pharyngitis only
B. Retropharyngeal abscess
C. Peritonsillar abscess
D. Oral candidiasis
E. Foreign body

Answer and topic summary

The answer is C. Peritonsillar abscess

A peritonsillar abscess is a collection of pus near the tonsils. The most common causes are Streptococcus pyogenes (group A), Streptococcus anginosus, & Staphylococcus aureus. Clinical features include muffled voice, sore throat, fever, drooling, neck pain, fatigue, and decreased PO intake. On physical exam, the uvula may be deviated. Diagnostic tests that can be done include ultrasound or CT with contrast (depends). Treatment is antibiotics, drainage, and supportive care.

Smarty PANCE Content Blueprint Review:

Covered under ⇒ PANCE Blueprint EENT ⇒ Oropharyngeal disorders ⇒ Infectious and inflammatory disorders ⇒ Peritonsillar abscess

Also covered as part of the Pediatric EORFamily Medicine EOR, and Emergency Medicine EOR topic list

5. Which of the following is the most common cause of acute epiglottitis?

A. Staphylococcus aureus
B. Haemophilus influenzae
C. Streptococcus pneumoniae
D. Streptococcus pyogenes
E. Neisseria meningitidis

Answer and topic summary

The answer is B. Haemophilus influenzae

Acute epiglottitis is an inflammatory condition of the epiglottis (usually due to a bacterial infection). In severe situations, it is life-threatening. The most common cause overall is said to be Haemophilus influenzae; however, in adults, Streptococcal species now tend to be more common. The management of this disease should focus on giving antibiotics and steroids. Airway management and securing an airway is the most important part of treatment.

Smarty PANCE Content Blueprint Review:

Covered under ⇒ PANCE Blueprint Pulmonary ⇒ Infectious Pulmonary Disorders ⇒ Acute epiglottitis

Also covered as part of the Pediatric EOR and Emergency Medicine EOR topic list

6. A 32-year-old female presents with pain around the nail fold on the index finger of her right hand. She is afebrile. On physical exam, you appreciate erythema and swelling of the proximal nail fold. The area is not fluctuant. Which of the following is the best treatment option?

A. Topical antibiotics and warm water soaks
B. Intravenous antibiotics
C. Oral antibiotics and incision and drainage
D. Surgical consult for removal
E. None of the above

Answer and topic summary

The answer is A. Topical antibiotics and warm water soaks

The patient has acute paronychia, which is a very common condition and essentially is just inflammation of the nail folds. The most common bacterial causes are Staph aureus and Strep pyogenes. Risk factors include manicures, nail biting, and picking at nails. Clinical features include sudden onset of painful erythema and swelling. Sometimes an abscess will be present. In this patient’s case, there was no fluctuance and so topical antibiotics and warm water soaks would be appropriate.

If the patient had an abscess, oral antibiotics and incision/drainage (with number 11 surgical blade) would be reasonable.

Smarty PANCE Content Blueprint Review:

Covered under ⇒ PANCE Blueprint Infectious DiseaseBacterial DiseaseMethicillin-resistant Staphylococcus aureus infection

7. A 60-year-old male presents with fatigue and weight loss. His physical exam is remarkable for gingival hyperplasia and splenomegaly. Labs reveal pancytopenia. A bone marrow biopsy reveals Auer rods. What is the likely diagnosis?

A. Acute myeloid leukemia
B. Acute lymphocytic leukemia
C. Chronic myeloid leukemia
D. Chronic lymphocytic leukemia
E. Non-Hodgkin’s lymphoma

Answer and topic summary

The answer is A. Acute myeloid leukemia

The patient has acute myeloid leukemia (AML), which is a group of cancers that involve the myeloid precursor cells. It is characterized by clonal proliferation (excessive growth) of abnormal myeloid precursor cells. Risk factors include smoking and chemotherapy/radiation.

Clinical features include fatigue, pallor, weakness, bone pain, gingival bleeding, and organomegaly. Lab findings are variable but may include pancytopenia (decrease in RBCs, WBCs, and platelets), electrolyte derangements, and/or hypoxemia. Workup should include a peripheral blood smear and bone marrow biopsy. The bone marrow biopsy will classically show Auer rods and >20% blasts.

Smarty PANCE Content Blueprint Review:

Covered under ⇒ PANCE Blueprint HematologyNeoplasms, premalignancies, and malignanciesAcute and chronic myelogenous leukemia

Also covered as part of the Family Medicine EOR topic list

8. A 42-year-old female presents for an annual physical exam with some complaints of fatigue and dyspnea. On physical exam, you appreciate a fixed S2 split. Which of the following is the most likely diagnosis?

A. Ventricular septal defect
B. Mitral regurgitation
C. Aortic stenosis
D. Atrial septal defect
E. Patent ductus arteriosus

Answer and topic summary

The answer is D. Atrial septal defect

Atrial septal defect is the most common congenital heart lesion in adults. People often don’t have symptoms until adulthood. The most common type is ostium secundum. Symptoms include fatigue and dyspnea. A classic physical exam finding on a test would be a systolic ejection crescendo-decrescendo flow murmur @ LUSB with FIXED S2, loud S1. Echocardiography is the initial imaging modality of choice. Keep in mind that a complication of ASD is that the left to right shunt can cause volume and pressure overload of the right heart and pulmonary circulation, leading to pulmonary hypertension.

Smarty PANCE Content Blueprint Review:

Covered under ⇒ PANCE Blueprint Cardiology ⇒ Congenital Heart Disease ⇒ Atrial septal defect

Also covered as part of the Pediatric EOR topic list

9. Which of the following is not a test for acute appendicitis?

A. Murphy sign
B. Rovsing sign
C. Obturator sign
D. Psoas sign
E. McBurney sign

Answer and topic summary

The answer is A. Murphy sign

Appendicitis is one of the most common indications for emergent abdominal surgery. The most common physical exam finding is RLQ tenderness. There are some tests that may help with the diagnosis — keep in mind though that these tests/signs are NOT sensitive.

  • Rovsing sign: pain in the RLQ with palpation of the LLQ (indicative of peritoneal irritation)
  • McBurney sign: tenderness about 2 inches from the ASIS (on a straight line from ASIS to the umbilicus)
  • Psoas sign: RLQ pain with passive right hip extension
  • Obturator sign: flexion of the right hip and knee, followed by internal rotation of the right hip elicits RLQ pain

Murphy’s sign is positive in acute cholecystitis, not appendicitis. A positive test is RUQ pain on inspiration.

Smarty PANCE Content Blueprint Review:

Covered under ⇒ PANCE Blueprint GI and Nutrition ⇒ Diseases of the Small Intestine ⇒ Appendicitis

Also covered as part of the Emergency Medicine EORFamily Medicine EORPediatric EOR, and General Surgery EOR topic list

10. A 31-year-old female presents with pain and numbness in her anterior tibial region. On physical exam, the area is pale, cold, and pulseless. Which of the following is the best definitive treatment for this likely diagnosis?

A. Antiplatelets
B. Amputation
C. Physical therapy
D. Fasciotomy
E. IV heparin drip

Answer and topic summary

The answer is D. Fasciotomy

The patient has acute compartment syndrome, which is when the tissue pressure within a closed muscle compartment exceeds the perfusion pressure. It can result in ischemia. The most common location is the calf. Many cases of acute compartment syndrome in the lower extremity are associated with fractures, burn injuries, crush injuries, or soft tissue infections. The “5 P’s” can help you remember the symptoms – painpallorparesthesiapulselessnessparalysis. Extremity fasciotomy is the treatment for acute compartment syndrome.

Smarty PANCE Content Blueprint Review:

Covered under ⇒ PANCE Blueprint Musculoskeletal  ⇒ Compartment Syndrome

This podcast is available on every device!

You can download and listen to past FREE episodes here, on iTunes, Spotify, Google Podcasts, Stitcher, Amazon Music, and all podcasting apps.

Download Interactive Content Blueprint Checklists for the PANCE, PANRE, EOR, and PANRE-LA

Interactive Content Blueprints for the PANCE PANRE and PANRE-LA

Follow this link to download your FREE copy of the PANCE/PANRE/EOR Content Blueprint Checklists.

Print it up and start crossing out the topics you understand, marking the ones you don’t, and making notes of key terms you should remember. The PDF version is interactive and linked directly to the individual lessons on Smarty PANCE.

The post Podcast Episode 102: Ten PANCE, PANRE, and Rotation Review Questions appeared first on The Audio PANCE and PANRE.

]]>
Listen to Podcast Episode 102: Ten PANCE, PANRE, and Rotation Review Questions If you can’t see the audio player, click here to listen to the full episode. Welcome to episode 102 of the Audio PANCE and PANRE Physician Assistant/Associate Board Review P... If you can’t see the audio player, click here to listen to the full episode.
Welcome to episode 102 of the Audio PANCE and PANRE Physician Assistant/Associate Board Review Podcast.
Join me today as we cover ten board review questions for your PANCE, PANRE, EOR™, and EOC™ exams.
Special from today’s episode:

* Read The New 2023 PANRE and PANRE-LA: Everything you Need to Know
* Members can try out the newly updated PANRE-LA Smart Search (you must log in to access the search bar)
* Sign up for the Entire Blueprint Email Series
* Follow Smarty PANCE and The Daily PANCE Blueprint on Instagram
* Follow Smarty PANCE and The Daily PANCE Blueprint on Facebook
* Join the Smarty PANCE Member’s Community then sign up for a study group to get updates about upcoming webinars.

I hope you enjoy this free audio component of the examination portion of this site. Smarty PANCE includes over 2,000 interactive board review questions, along with flashcards, ReelDx cases, integrated Picmonics, and lessons covering every blueprint topic available to all Smarty PANCE members.

* You can download and listen to past FREE episodes here, on iTunes, Spotify, Google Podcasts, Stitcher, and most podcasting apps.
* You can listen to all the latest episodes, take interactive quizzes, and download more resources on each episode page.

Interactive exam to complement today’s podcast
1. Which of the following is NOT true about a non-ST elevation myocardial infarction?
A. Non-enteric-coated, chewable aspirin 325 mg should be given
B. Troponins are elevated
C. It happens due to a partially occluded epicardial coronary artery
D. Patients need a 12-lead EKG
E. You will always see ST depressions
2. A 22-year-old G1P0 female at 28 weeks gestation with a history of diabetes presents to the clinic with fever, chills, and dysuria. Vitals show tachycardia (115 bpm), tachypnea (22 bpm), and hypotension (90/58 mmHg). Physical exam reveals suprapubic tenderness. Labs reveal leukocytosis, hyponatremia, and hyperglycemia. Urine dipstick is positive for nitrites, blood, glucose, and ketones. Which of the following is the next best step?
A. Send home with oral antibiotics and analgesics
B. Reassurance and oral rehydration solution
C. Admit to hospital for antibiotics, fluids, & insulin
D. Order outpatient CT scan of the abdomen
E.]]>
The Physician Assistant Life | Smarty PANCE full 21:57 556
Podcast Episode 101: Ten PANCE, PANRE, and Rotation Review Questions http://podcast.thepalife.com/podcast-episode-101-ten-pance-panre-and-rotation-review-questions/ Wed, 02 Nov 2022 04:26:35 +0000 https://podcast.thepalife.com/?p=535 Welcome to episode 101 of the Audio PANCE and PANRE Physician Assistant/Associate Board Review Podcast. Join me today as we cover ten board review questions for your PANCE, PANRE, and rotation exams. Special from today’s episode: Join the Smarty PANCE Member’s Community then sign up for a study group to get updates about upcoming webinars. […]

The post Podcast Episode 101: Ten PANCE, PANRE, and Rotation Review Questions appeared first on The Audio PANCE and PANRE.

]]>
Podcast Episode 101 - Ten Questions For Your PANCE PANRE and EOC EOR ExamsWelcome to episode 101 of the Audio PANCE and PANRE Physician Assistant/Associate Board Review Podcast.

Join me today as we cover ten board review questions for your PANCE, PANRE, and rotation exams.

Special from today’s episode:

I hope you enjoy this free audio component of the examination portion of the Smarty PANCE website. The full board review website includes over 2,000 interactive board review questions, flashcards, and blueprint lessons available to all members of Smarty PANCE.

Listen to Podcast Episode 101: Ten PANCE, PANRE, and Rotation Review Questions

If you can’t see the audio player, click here to listen to the full episode.

Interactive exam to complement today’s podcast

1. A 75-year-female smoker with a history of atrial fibrillation and hypertension presents to the ER complaining of a 2-hour history of right-sided weakness and aphasia that has now resolved. Her physical exam and vital signs are completely unremarkable. CT head is unremarkable. Which of the following is the most likely diagnosis?

A. Subarachnoid hemorrhage
B. Transient ischemic attack
C. Cerebral venous sinus thrombosis
D. Multiple sclerosis
E. Complicated migraine

Answer and topic summary

The answer is B. Transient ischemic attack

The patient had a transient ischemic attack (TIA), which is characterized by transient neurological symptoms without objective evidence of acute infarction. Symptoms vary, but patients may have hemiparesis, hemiplegia, aphasia, or vision loss. Risk factors include alcohol, hypertension, smoking, diabetes, illicit drug use, and atrial fibrillation.

Urgent evaluation is needed in patients with symptoms of TIA (e.g., coagulation studies, TTE, EKG, CT or MRI head, imaging of the cervicocephalic vasculature via carotid US, CTA, or MRA). Treatment for high-risk patients includes dual antiplatelet therapy (ASA + clopidogrel) for 21 days and risk factor management (statin, exercise, etc). Remember the risk of an actual stroke is high after a TIA.

View blueprint lesson

Smarty PANCE Content Blueprint Review:

Covered under ⇒ PANCE Blueprint NeurologyVascular DisorderTransient ischemic attack

Also covered as part of the Internal Medicine EOR, Family Medicine EOR, and Emergency Medicine EOR topic list

2. A 26-year-old female presents with a history of miscarriages and recurrent pulmonary embolism. She is diagnosed with antiphospholipid syndrome. Which of the following is the mainstay of treatment for this condition?

A. Dual antiplatelet therapy (DAPT)
B. Aspirin and heparin
C. Dabigatran
D. Warfarin
E. Heparin

Answer and topic summary

The answer is D. Warfarin

Antiphospholipid syndrome (APS) is an autoimmune disease defined by venous thromboembolism, arterial thrombosis, and obstetric morbidity in the presence of circulating antiphospholipid antibodies (aPLs). It is the most common form of acquired thrombophilia.

aPLs include lupus anticoagulant, anticardiolipin, anti-b2-glycoprotein I antibodies. Clinical features include DVTs (32%), thrombocytopenia (22%), livedo reticularis (20%), stroke (13%), PEs (9%), and fetal loss (8%). The mainstay of treatment is warfarin. Other anticoagulants have been found to be less effective than warfarin so far.

View blueprint lesson

Smarty PANCE Content Blueprint Review:

Covered under ⇒ PANCE Blueprint HematologyCoagulation DisordersHypercoagulable states

Also covered as part of the Internal Medicine EOR and Emergency Medicine EOR topic list

3. Which of the following is the most common cause of acute bronchitis in the United States?

A. Virus
B. Bacteria
C. Fungus
D. Allergies
E. Tuberculosis

Answer and topic summary

The answer is A. Virus

Acute bronchitis is characterized by a cough due to inflammation of the large airways and trachea with no evidence of pneumonia. Acute bronchitis often follows an upper respiratory infection. The most common cause of acute bronchitis is viral. Symptoms include cough (10-20 days), headache, shortness of breath, and wheezing. The presence of fever should make you consider pneumonia or influenza.

Diagnosis is clinical. A chest radiograph should be given if it is hard to clinically distinguish between pneumonia and bronchitis. Treatment is mainly supportive care (rest, hydration) and symptomatic management (antitussives for cough, bronchodilators for wheezing, etc). Antibiotics should NOT be given for acute bronchitis.

View blueprint lesson

Smarty PANCE Content Blueprint Review:

Covered under ⇒ PANCE Blueprint PulmonaryInfectious Pulmonary DisordersAcute bronchitis

Also covered as part of the Emergency Medicine EOR topic list

4. A 27-year-old male presents to the clinic with shortness of breath, chest pain, and fatigue. His physical exam is remarkable for a crescendo-decrescendo systolic murmur heard at the apex. An echocardiogram reveals asymmetric septal hypertrophy and left ventricular hypertrophy. Which of the following is the best initial treatment option?

A. Procainamide
B. Hydrochlorothiazide
C. Beta-blockers
D. Aspirin
E. ACE inhibitor

Answer and topic summary

The answer is C. Beta-blockers

The patient has hypertrophic cardiomyopathy (HCM), which is a genetic disease of the heart muscle due to mutations in the sarcomere genes. It often presents with fatigue, dyspnea, chest pain, or syncope. The murmur on the physical exam is due to LV obstruction; it will be a harsh crescendo-decrescendo systolic murmur heard at the apex and LLSB. It’s important to keep in mind that the murmur intensity will decrease with more venous return (squatting) and increase with less venous return (Valsalva).

Diagnostic tests include an EKG, echocardiogram, and exercise stress testing. Treatment is indicated for symptomatic patients and includes a negative inotropic agent (i.e., nondihydropyridine CCBs or beta-blockers) and diuretics as needed for volume overload. If patients are refractory to medications, septal myectomy and percutaneous septal ablation can be performed.

View blueprint lesson

Smarty PANCE Content Blueprint Review:

Covered under ⇒ PANCE Blueprint CardiologyCardiomyopathyHypertrophic Cardiomyopathy

Also covered as part of the Pediatric Rotation EOR topic list

5. A 22-year-old female presents to the ER with crampy lower abdominal pain and vaginal bleeding for the past 2 hours. She missed her last menstrual cycle. The physical exam is remarkable for an open cervical os. No products of conception are observed. Which of the following is the most likely diagnosis?

A. Threatened abortion
B. Incomplete abortion
C. Missed abortion
D. Septic abortion
E. Inevitable abortion

Answer and topic summary

The answer is E. Inevitable abortion

The patient most likely has a spontaneous abortion, which is a nonviable intrauterine pregnancy up to 20 weeks gestation. The most common cause of miscarriages in the first trimester is chromosomal abnormalities. There are many different types of spontaneous abortions.

The patient specifically has an inevitable abortion, which typically presents with vaginal bleeding, crampy abdominal pain, and an open external cervical os without expulsion of products of conception. The fetus is not viable.

View blueprint lesson

Smarty PANCE Content Blueprint Review:

Covered under ⇒ PANCE Blueprint Reproductive SystemComplicated PregnancyAbortion

Also covered as part of the Women’s Health EOR topic list

6. Which of the following is not a risk factor for Methicillin-resistant Staphylococcus aureus (MRSA)?

A. Previous antibiotic use
B. Injection drug use
C. Indwelling hemodialysis catheter
D. Presence in long-term facility
E. All of the above are risk factors

Answer and topic summary

The answer is E. All of the above are risk factors 

Methicillin-resistant Staphylococcus aureus (MRSA) infection is caused by a certain type of S. aureus that is resistant to several antibiotics. Risk factors include previous antibiotic use, injection drug use, indwelling hemodialysis catheter, living in a long-term care facility, HIV infection, and recent hospitalization.

The most common site for MRSA infections to occur is the skin/soft tissue. Oral antibiotics that cover MRSA include Bactrim, doxycycline, and clindamycin. For severe MRSA infections, IV vancomycin is preferred.

View blueprint lesson

Smarty PANCE Content Blueprint Review:

Covered under ⇒ PANCE Blueprint Infectious DiseaseBacterial DiseaseMethicillin-resistant Staphylococcus aureus infection

7. A 60-year-old male presents with fatigue and weight loss. His physical exam is remarkable for gingival hyperplasia and splenomegaly. Labs reveal pancytopenia. A bone marrow biopsy reveals Auer rods. What is the likely diagnosis?

A. Acute myeloid leukemia
B. Acute lymphocytic leukemia
C. Chronic myeloid leukemia
D. Chronic lymphocytic leukemia
E. Non-Hodgkin’s lymphoma

Answer and topic summary

The answer is A. Acute myeloid leukemia

The patient has acute myeloid leukemia (AML), which is a group of cancers that involve the myeloid precursor cells. It is characterized by clonal proliferation (excessive growth) of abnormal myeloid precursor cells. Risk factors include smoking and chemotherapy/radiation.

Clinical features include fatigue, pallor, weakness, bone pain, gingival bleeding, and organomegaly. Lab findings are variable but may include pancytopenia (decrease in RBCs, WBCs, and platelets), electrolyte derangements, and/or hypoxemia. The workup should include a peripheral blood smear and bone marrow biopsy. The bone marrow biopsy will classically show Auer rods and >20% blasts.

View blueprint lesson

Smarty PANCE Content Blueprint Review:

Covered under ⇒ PANCE Blueprint HematologyNeoplasms, premalignancies, and malignanciesAcute and chronic myelogenous leukemia

Also covered as part of the Internal Medicine EOR, Emergency Medicine EOR, Family Medicine EOR, and Pediatric EOR topic list

8. Giant Cell Arteritis: The Daily PANCE Blueprint

A 55-year-old female presents with right-sided headaches, jaw pain upon chewing, and mild dizziness for the past few months. On physical exam, the right temporal artery is tender to palpation. Labs reveal an elevated ESR and CRP. What is the definitive diagnosis for this condition?

A. CT head
B. Temporal artery biopsy
C. PET scan
D. Ultrasound
E. IgA antibody level

Answer and topic summary

The answer is B. Temporal artery biopsy

The patient has temporal arteritis, which is a vasculitis of large and medium vessels. Clinical features often include fever, fatigue, weight loss, headache, jaw claudication, and transient vision loss. The typical clinical vignette is an older female presenting with a headache and jaw claudication. On physical exam, you may appreciate a temporal artery that is thickened, tender, or erythematous.

ESR and CRP are sensitive, but not specific for the diagnosis. The definitive diagnosis of temporal arteritis is a temporal artery biopsy. Treatment is high-dose steroids. It’s important that temporal arteritis is treated because it can lead to blindness, thrombotic events, and other complications.

View blueprint lesson

Smarty PANCE Content Blueprint Review:

Covered under ⇒ PANCE Blueprint CardiologyVascular DiseaseGiant cell arteritis

Also covered as part of the Internal Medicine EOR topic list

9. Which of the following is one of the most common initial clinical symptoms in multiple sclerosis?

A. Vertigo
B. Extreme weakness
C. Depression
D. Arthralgias
E. Optic neuritis

Answer and topic summary

The answer is E. Optic neuritis

Multiple sclerosis (MS) is a demyelinating disease of the central nervous system. There are many types of MS patterns. Clinical features suggestive of MS include sensory loss in limbs or one side of the face, gait/balance issues, heat sensitivity (Uhthoff), fatigue, and Lhermitte sign. The most common presenting symptom is optic neuritis.

MRI of the brain (+/- spinal cord) is the diagnostic test of choice. The McDonald diagnostic criteria can be used to help make the diagnosis of MS. Treatment of MS includes disease-modifying pharmacotherapy, such as glatiramer acetate (Copaxone), dimethyl fumarate, natalizumab (Tysabri), interferons, and many more. Acute exacerbations of MS are managed with short-term, high-dose glucocorticoids.

View blueprint lesson

Smarty PANCE Content Blueprint Review:

Covered under ⇒ PANCE Blueprint NeurologyNeuromuscular disordersMultiple sclerosis

Also covered as part of the Internal Medicine EOR topic list

10. A 52-year-old male presents to the clinic complaining of “sharp burning” in his lower chest after eating a large meal. The pain goes away when he raises the head of the bed. Which of the following is the best pharmacological agent for this likely diagnosis?

A. Esomeprazole
B. Nitroglycerin
C. Aspirin
D. Codeine
E. Sucralfate

Answer and topic summary

The answer is A. Esomeprazole

The patient has gastrointestinal reflux disease (GERD), which is a common condition characterized by stomach acid flowing up into the esophagus. Classic symptoms include heartburn (burning sensation around the sternum) after eating, water brash, regurgitation, and dysphagia.

The diagnosis is usually clinical; however, if the patient has alarming symptoms (e.g., weight loss, anemia, etc), it is appropriate to get an upper endoscopy. The gold standard is an ambulatory 24-hour pH monitoring system. Initial management includes weight loss, elevating the head of the bed, and eliminating triggers (spicy foods, chocolate, etc). Most recommend starting with histamine 2 receptor antagonist therapy first (e.g., ranitidine) and then transitioning to proton pump inhibitors (e.g., esomeprazole).

View blueprint lesson

Smarty PANCE Content Blueprint Review:

Covered under ⇒ PANCE Blueprint GI and NutritionEsophageal DisordersGastroesophageal reflux disease

Also covered as part of the Family Medicine EOR, Internal Medicine EOR, and Pediatric EOR topic list

This podcast is available on every device!

You can download and listen to past FREE episodes here, on iTunes, Spotify, Google Podcasts, Stitcher, Amazon Music, and all podcasting apps.

Download Interactive Content Blueprint Checklists for the PANCE, PANRE, EOR, and PANRE-LA

Interactive Content Blueprints for the PANCE PANRE and PANRE-LA

Follow this link to download your FREE copy of the PANCE/PANRE/EOR Content Blueprint Checklists.

Print it up and start crossing out the topics you understand, marking the ones you don’t, and making notes of key terms you should remember. The PDF version is interactive and linked directly to the individual lessons on Smarty PANCE.

The post Podcast Episode 101: Ten PANCE, PANRE, and Rotation Review Questions appeared first on The Audio PANCE and PANRE.

]]>
Welcome to episode 101 of the Audio PANCE and PANRE Physician Assistant/Associate Board Review Podcast. Join me today as we cover ten board review questions for your PANCE, PANRE, and rotation exams. Special from today’s episode: Join the Smarty PANCE ... Welcome to episode 101 of the Audio PANCE and PANRE Physician Assistant/Associate Board Review Podcast.
Join me today as we cover ten board review questions for your PANCE, PANRE, and rotation exams.
Special from today’s episode:

* Join the Smarty PANCE Member’s Community then sign up for a study group to get updates about upcoming webinars.
* Check out our updated End of Curriculum™ (EOC) Exam Course
* Sign up for the Entire Blueprint Email Series
* Follow Smarty PANCE and The Daily PANCE Blueprint on Instagram
* Follow Smarty PANCE and The Daily PANCE Blueprint on Facebook

I hope you enjoy this free audio component of the examination portion of the Smarty PANCE website. The full board review website includes over 2,000 interactive board review questions, flashcards, and blueprint lessons available to all members of Smarty PANCE.

* You can download and listen to past FREE episodes here, on iTunes, Spotify, Google Podcasts, Stitcher, and most podcasting apps.
* You can listen to all the latest episodes, take interactive quizzes, and download more resources on each episode page.

Listen to Podcast Episode 101: Ten PANCE, PANRE, and Rotation Review Questions
If you can’t see the audio player, click here to listen to the full episode.
Interactive exam to complement today’s podcast
1. A 75-year-female smoker with a history of atrial fibrillation and hypertension presents to the ER complaining of a 2-hour history of right-sided weakness and aphasia that has now resolved. Her physical exam and vital signs are completely unremarkable. CT head is unremarkable. Which of the following is the most likely diagnosis?
A. Subarachnoid hemorrhage
B. Transient ischemic attack
C. Cerebral venous sinus thrombosis
D. Multiple sclerosis
E. Complicated migraine
2. A 26-year-old female presents with a history of miscarriages and recurrent pulmonary embolism. She is diagnosed with antiphospholipid syndrome. Which of the following is the mainstay of treatment for this condition?
A. Dual antiplatelet therapy (DAPT)
B. Aspirin and heparin
C. Dabigatran
D. Warfarin
E. Heparin
3. Which of the following is the most common cause of acute bronchitis in the United States?
A. Virus
B. Bacteria
C. Fungus
D. Allergies
E. Tuberculosis
4. A 27-year-old male presents to the clinic with shortness of breath, chest pain, and fatigue. His physical exam is remarkable for a crescendo-decrescendo systolic murmur heard at the apex.]]>
The Physician Assistant Life | Smarty PANCE full 20:21 535
Podcast Episode 100: Atrial Fibrillation for the PANCE and PANRE http://podcast.thepalife.com/episode-100/ Tue, 20 Sep 2022 18:15:06 +0000 http://podcast.thepalife.com/?p=523 Welcome to episode one hundred of the Audio PANCE and PANRE Physician Assistant/Associate Board Review Podcast. Join me today as we cover atrial fibrillation for the PANCE, PANRE, and EOR™ exams. Special from today’s episode: Join the Smarty PANCE Member’s Community then sign up for the Sunday Funday Study Group. Once you have signed up, […]

The post Podcast Episode 100: Atrial Fibrillation for the PANCE and PANRE appeared first on The Audio PANCE and PANRE.

]]>
Welcome to episode one hundred of the Audio PANCE and PANRE Physician Assistant/Associate Board Review Podcast.

Join me today as we cover atrial fibrillation for the PANCE, PANRE, and EOR™ exams.

Special from today’s episode:

Below you will find the audio of today’s podcast.

The Audio PANCE/PANRE and EOR PA Board Review Podcast

I hope you enjoy this free audio component to the examination portion of this site. The full board review course includes over 2,000 interactive board review questions and is available to all members of Smarty PANCE.

Listen to Podcast Episode 100: Atrial Fibrillation for the PANCE and PANRE

If you can’t see the audio player, click here to listen to the full episode.

The post Podcast Episode 100: Atrial Fibrillation for the PANCE and PANRE appeared first on The Audio PANCE and PANRE.

]]>
Welcome to episode one hundred of the Audio PANCE and PANRE Physician Assistant/Associate Board Review Podcast. Join me today as we cover atrial fibrillation for the PANCE, PANRE, and EOR™ exams. Special from today’s episode: Join the Smarty PANCE Memb... Welcome to episode one hundred of the Audio PANCE and PANRE Physician Assistant/Associate Board Review Podcast.
Join me today as we cover atrial fibrillation for the PANCE, PANRE, and EOR™ exams.
Special from today’s episode:

* Join the Smarty PANCE Member’s Community then sign up for the Sunday Funday Study Group. Once you have signed up, you can access the Webinar replay of this episode here
* Check out our updated End of Curriculum™ (EOC) Exam Course (now with orthopedics!)
* Sign up for the Entire Blueprint email series
* Follow Smarty PANCE and The Daily PANCE Blueprint on Instagram
* Follow Smarty PANCE and The Daily PANCE Blueprint on Facebook

Below you will find the audio of today’s podcast.
The Audio PANCE/PANRE and EOR PA Board Review Podcast
I hope you enjoy this free audio component to the examination portion of this site. The full board review course includes over 2,000 interactive board review questions and is available to all members of Smarty PANCE.

* You can download and listen to past FREE episodes here, on iTunes, Spotify, Google Podcasts, Stitcher, and most podcasting apps.
* You can listen to all the latest episodes, take interactive quizzes, and download more resources on each episode page.

Listen to Podcast Episode 100: Atrial Fibrillation for the PANCE and PANRE
If you can’t see the audio player, click here to listen to the full episode.
]]>
The Physician Assistant Life | Smarty PANCE full 59:06 523
Podcast Episode 99: Ten PANCE, PANRE, and Rotation Review Questions http://podcast.thepalife.com/podcast-episode-99-ten-pance-panre-and-rotation-review-questions/ Thu, 21 Jul 2022 14:08:30 +0000 http://podcast.thepalife.com/?p=505 Welcome to episode 99 of the Audio PANCE and PANRE Physician Assistant/Associate Board Review Podcast. Join me as I cover ten PANCE, PANRE, and EOR™ review questions from the Smarty PANCE Instagram/Facebook page and the smartypance.com board review website. Special from today’s episode: Join the Smarty PANCE Member’s Community Check out our all-new End of […]

The post Podcast Episode 99: Ten PANCE, PANRE, and Rotation Review Questions appeared first on The Audio PANCE and PANRE.

]]>
The Audio PANCE and PANRE Episode 99Welcome to episode 99 of the Audio PANCE and PANRE Physician Assistant/Associate Board Review Podcast.

Join me as I cover ten PANCE, PANRE, and EOR™ review questions from the Smarty PANCE Instagram/Facebook page and the smartypance.com board review website.

Special from today’s episode:

Below you will find an interactive exam to complement today’s podcast.

The Audio PANCE/PANRE and EOR PA Board Review Podcast

I hope you enjoy this free audio component to the examination portion of this site. The full board review course includes over 2,000 interactive board review questions and is available to all members of Smarty PANCE.

Listen Carefully Then Take the Practice Exam

If you can’t see the audio player, click here to listen to the full episode.

Podcast Episode 99: Ten PANCE/PANRE and EOR Topic Blueprint Questions

1. A 42-year-old male on lithium presents with polyuria, nocturia, and polydipsia. Laboratory findings are remarkable for slightly elevated sodium. Which of the following is the most likely diagnosis?

A. Neurogenic diabetes insipidus
B. Nephrogenic diabetes insipidus
C. Type 2 diabetes mellitus
D. SIADH
E. Adrenal insufficiency

Click here to see the answer

The answer is B. Nephrogenic diabetes insipidus

Nephrogenic diabetes insipidus (DI) is a condition where the kidneys are resistant to the effects of ADH. Nephrogenic DI can occur secondary to lithium toxicity or chronic lithium usepregnancyinherited disorders, and electrolyte issues. Clinical features include polyuria, nocturia, and polydipsia. Serum sodium is either normal or high.

In this patient, lithium has entered the collecting duct, accumulated, and interfered with ADH’s capacity to increase water permeability. Treatment options for this patient include amiloride or thiazide diuretics. In general, whether or not lithium is discontinued depends on many factors.

VIEW BLUEPRINT LESSON

Smarty PANCE Content Blueprint Review:

Covered under ⇒ PANCE Blueprint Endocrinology ⇒ Pituitary Disorders ⇒ Diabetes insipidus

Also covered as part of the Internal Medicine EOR and Emergency Medicine PAEA EOR topic list

2. A 50-year-old female presents with poor appetite, low energy, poor concentration, and feelings of hopelessness on most days for the past 3 years. She denies suicidal ideation. She has never had a past manic or hypomanic episode. Which of the following is the best treatment option?

A. Haloperidol
B. Fluoxetine
C. Lorazepam
D. Amitriptyline
E. Risperidone

Click here to see the answer

The answer is B. Fluoxetine

This patient has dysthymia, or persistent depressive disorder. The DSM V criteria follow:

  • Depressed mood ≥ 2 years on most days
  • At least 2 of the following: appetite changes, sleep changes, low energy, low self-esteem, poor concentration, hopelessness
  • Not without symptoms > 2 months at a time
  • No mania or hypomania episodes, ever

Like other psychiatric disorders, symptoms can’t be attributable to drugs, and the symptoms must cause distress/impairment. 1st line treatment is selective serotonin reuptake inhibitors and psychotherapy.

VIEW BLUEPRINT LESSON

Smarty PANCE Content Blueprint Review:

Covered under ⇒ PANCE Blueprint Psychiatry ⇒ Depressive disorders ⇒ Persistent depressive disorder (dysthymia)

Also covered as part of the Psychiatry EOREmergency Medicine EOR, and Pediatric PAEA EOR topic list

3. An 85-year-old male with a history of chronic kidney disease presents to the ER with muscle cramps. Laboratory studies reveal potassium of 7.8 mEq/L. EKG reveals peaked T waves. Which of the following is the best initial med to give?

A. Insulin
B. Albuterol
C. Furosemide
D. Sodium bicarbonate
E. Calcium gluconate

Click here to see the answer

The answer is E. Calcium gluconate

Hyperkalemia is caused by many things: iatrogenic (ACEI/ARBs), cellular destruction (hemolysis, tumor lysis syndrome, burns), renal failure, adrenal insufficiency, etc. Symptoms are nonspecific and include muscle weakness, N/V, decreased DTRs, etc. Classically on EKG, you may see peaked T waves (there are many other findings though…such as QRS widening, PR interval prolongation, sine-wave, etc). Management for severe hyperkalemia includes calcium gluconate (stabilizes the resting membrane potential of the myocardial membrane), shifting potassium intracellularly (via beta-agonists, sodium bicarbonate, insulin), and removing potassium (via diuretics, kayexalate, or hemodialysis).

VIEW BLUEPRINT LESSON

Smarty PANCE Content Blueprint Review:

Covered under ⇒ PANCE Blueprint Renal SystemFluid and Electrolyte DisordersHyperkalemia/hypokalemia

Also covered as part of the Emergency Medicine EOR and General Surgery PAEA EOR topic list

4. A 28-year-old male with sickle cell disease presents to the ER with chest pain, dyspnea, and a cough for the past day. Vitals are remarkable for SpO2 91% and T 102.2F. A CXR reveals bilateral pulmonary infiltrates. On physical exam the patient is alert, speaking in full sentences, and breathing without accessory muscle use. Which of the following is the most appropriate management option for this patient?

A. Surgical consultation, antibiotics, plasmapheresis, IVIG
B. Pain control, hydration, blood transfusions, oxygen, antibiotics, VTE prophylaxis
C. Immediate endotracheal intubation, aggressive intravenous fluids, antibiotics, blood transfusions
D. Blood transfusions and oxygen only
E. Empiric antibiotics and oxygen only

Click here to see the answer

The answer is B. Pain control, hydration, blood transfusions, oxygen, antibiotics, VTE prophylaxis

The patient has acute chest syndrome (ACS), which is defined as a new radiodensity on CXR with fever and/or pulmonary symptoms. It occurs due to vaso-occlusion in the pulmonary microvasculature, subsequently leading to deoxygenation of hemoglobin and sickling of RBCs. ACS is the main cause of death in patients with sickle cell disease (SCD). About 1 in 2 patients with SCD get ACS.

Patients may present with chest pain, extremity pain, and shortness of breath. Causes of ACS include fat emboli, infection, asthma, oversedation, and post-op issues. Treatment typically includes fluidsoxygenpain controlblood transfusionsantibiotics, and VTE prophylaxis.

VIEW BLUEPRINT LESSON

Smarty PANCE Content Blueprint Review:

Covered under ⇒ PANCE Blueprint Hematology ⇒ Hemoglobinopathies ⇒ Sickle cell anemia

Also covered as part of the Internal Medicine EOR and Emergency Medicine PAEA EOR topic list

5. Which of the following is the most common cause of bacterial sialadenitis?

A. Bacteroides
B. Staphylococcus aureus
C. Streptococcus pneumoniae
D. Escherichia coli
E. Streptococcus viridans

Click here to see the answer

The answer is B. Staphylococcus aureus

Bacterial sialadenitis is defined by inflammation of a salivary gland due to a bacterial infection. It is most commonly caused by Staphylococcus aureusUsually, bacterial sialadenitis happens in the setting of a salivary gland stone (reduced salivary flow leads to bacteria building up in the mouth). Other risk factors include bad oral hygiene and old age. The most common gland affected is the parotid gland.

Clinical features include acute onset of fever, chills, and swelling/tenderness of the affected gland (possible to see purulent drainage). The treatment is broad-spectrum antibiotics: IV ampicillin-sulbactam. Surgical drainage is needed if an abscess develo

VIEW BLUEPRINT LESSON

Smarty PANCE Content Blueprint Review:

Covered under ⇒ PANCE Blueprint EENT ⇒ Salivary disorders ⇒ Sialadenitis

Also covered as part of the Family Medicine PAEA EOR topic list

6. A 52-year-old female presents to the clinic complaining of chronic bone pain, constipation, and fatigue. Her last lab results reveal a decline in the glomerular filtration rate. Which of the following is the most likely diagnosis?

A. Acute myelogenous leukemia
B. Bronchogenic carcinoma
C. Multiple myeloma
D. Polymyalgia rheumatica
E. Colorectal cancer

Click here to see the answer

The answer is C. Multiple myeloma

The patient has multiple myeloma, which is a clonal proliferation of abnormal plasma cells. It can present with “BREAK” symptoms: Bone pain, Recurrent infections, Elevated calcium, Anemia, and Kidney failure. A serum and urine protein electrophoresis will reveal a monoclonal spike. Other diagnostic studies include a CT or MRI (reveals lytic bone lesions), peripheral blood smear (normocytic anemia in rouleaux formation), and urinalysis (Bence Jones proteins aka free light chains).

The definitive diagnosis is a bone marrow biopsy. Treatment is high-dose chemotherapy with autologous hematopoietic cell transplantation.

VIEW BLUEPRINT LESSON

Smarty PANCE Content Blueprint Review:

Covered under ⇒ PANCE Blueprint HematologyNeoplasms, premalignancies, and malignanciesMultiple myeloma

Also covered as part of the Internal Medicine PAEA EOR topic list

7. A 56-year-old post-menopausal G0P0 female presents to the clinic complaining of abnormal uterine bleeding and weight loss for the past 5 months. Physical exam is unremarkable. Which of the following is not a risk factor for this patient’s likely diagnosis?

A. Nulliparity
B. Tamoxifen
C. Chronic anovulation
D. Obesity
E. Late menarche

Click here to see the answer

The answer is E. Late menarche

The patient has endometrial cancer, the most commonly encountered gynecologic cancer. The cardinal symptom is abnormal uterine bleeding. Other clinical features include pelvic pain, pelvic masses, and weight loss.

Risk factors include conditions or medications that lead to chronically increased estrogen levels, such as nulliparity, tamoxifen, anovulation, and obesity. Early menarche and late menopause would be risk factors, not late menarche.

VIEW BLUEPRINT LESSON

Smarty PANCE Content Blueprint Review:

Covered under ⇒ PANCE Blueprint Reproductive SystemNeoplasms of the breast and reproductive tractEndometrial cancer

Also covered as part of the Women’s Health PAEA EOR topic list

8. Which of the following are the two most common etiologies of peptic ulcer disease?

A. H. pylori and NSAIDs
B. Spicy foods and acute stress
C. Chemical ingestion and GERD
D. Smoking and alcohol use
E. Chronic stress and radiation

Click here to see the answer

The answer is A. H. pylori and NSAIDs

Peptic ulcer disease (PUD) is a common GI disease defined by a defect in the mucosal lining of the stomach or duodenum. The two most common causes are H. pylori and NSAIDs. Other more rare culprits include Zollinger-Ellison Syndrome, cancer, stress, and radiation. Surprisingly, patients are asymptomatic about 70% of the time. Common symptoms are epigastric abdominal paindyspepsia, and bloating. Overall, an upper endoscopy is the best diagnostic test. A biopsy should be done for ulcers with malignant features on all gastric ulcers.

The preferred treatment is PPIs. If H. pylori is the cause, an antibiotic regimen is warranted (e.g., amoxicillin, clarithromycin, and a PPI).

VIEW BLUEPRINT LESSON

Smarty PANCE Content Blueprint Review:

Covered under ⇒ PANCE Blueprint GI and Nutrition ⇒ Gastric Disorders ⇒ Peptic ulcer disease

Also covered as part of the Internal Medicine EOREmergency Medicine EORFamily Medicine EOR, and General Surgery EOR topic list

9. A 72-year-old male smoker with a history of cancer presents to the clinic for an annual physical. Vitals are unremarkable. On physical exam, you notice erythema along the course of a superficial vein on his left leg. The area is mildly tender to palpation. The left leg is also larger than the other leg. Which of the following is the next best step?

A. Treat with NSAIDs and warm compresses
B. Order a duplex ultrasound
C. Initial anticoagulation immediately
D. Admit to the hospital
E. Reassurance and send home

Click here to see the answer

The answer is B. Order a duplex ultrasound

The patient has superficial thrombophlebitis, which is inflammation involving a superficial vein along with the presence of a superficial clot. Risk factors include varicose veins, estrogen, prior DVT, cancer, and hypercoagulable states. Clinical features include tendernessindurationpain, and erythematous skin over a superficial vein.

In many cases, duplex ultrasound is needed to rule out a concurrent DVT (studies show that up to 25-50% of patients with superficial thrombophlebitis have a concurrent DVT). Treatment includes NSAIDs, warm/cold compressesextremity elevation, and compression therapy. The decision to initiate anticoagulation for superficial thrombophlebitis depends on many factors.

VIEW BLUEPRINT LESSON

Smarty PANCE Content Blueprint Review:

Covered under ⇒ PANCE Blueprint Cardiology ⇒ Vascular Disease ⇒ Phlebitis/thrombophlebitis

10. A 63-year-old male with a history of hepatitis C presents with a pruritic rash located on the flexor surfaces of his wrist. On physical exam, you observe purplish papules with flat-tops. Which of the following is the most likely diagnosis?

A. Atopic dermatitis
B. Psoriasis
C. Lichen planus
D. Pemphigus vulgaris
E. Scabies

Click here to see the answer

The answer is C. Lichen planus

Lichen planus is a skin disorder characterized by a rash associated with the 5 Pspolygonal, pruritic, papules, plaques, and purple. It commonly affects the wrists and ankles. Lichen planus can also cause the classic Wickham’s striae, which are white-lacelike lines (usually in the oral mucosa). A risk factor is hepatitis C.

skin biopsy can help confirm the diagnosis. Treatment includes high potency topical corticosteroids. If a patient has an extensive form of lichen planus, then phototherapy, acitretin, or systemic steroids may help.

VIEW BLUEPRINT LESSON

Smarty PANCE Content Blueprint Review:

Covered under ⇒ PANCE Blueprint Dermatology ⇒ Papulosquamous Disorders ⇒ Lichen planus

Also covered as part of the Family Medicine EOR and Pediatric EOR topic list

Looking for all the podcast episodes?

This FREE podcast series is limited to every other episode, you can download and enjoy the complete audio series by becoming a Smarty PANCE member.

I will be releasing new episodes every few weeks. Smarty PANCE is now discounted, so sign up now before it’s too late!

Additional resources and links from the show

This Podcast is available on iOS and Android

Download the Interactive Content Blueprint Checklist

Interactive Content Blueprint for the 2022-2023 PANCE

Follow this link to download your FREE copy of the PANCE/PANRE/EOR™ Content Blueprint Checklists

Print it up and start crossing out the topics you understand, marking the ones you don’t, and making notes of key terms you should remember. The PDF version is interactive and linked directly to the individual lessons on Smarty PANCE.

Smarty PANCE is not sponsored or endorsed by, or affiliated with, the NCCPA or the Physician Assistant Education Association (PAEA). All trademarks are the property of their respective owners.

The post Podcast Episode 99: Ten PANCE, PANRE, and Rotation Review Questions appeared first on The Audio PANCE and PANRE.

]]>
Welcome to episode 99 of the Audio PANCE and PANRE Physician Assistant/Associate Board Review Podcast. Join me as I cover ten PANCE, PANRE, and EOR™ review questions from the Smarty PANCE Instagram/Facebook page and the smartypance. Welcome to episode 99 of the Audio PANCE and PANRE Physician Assistant/Associate Board Review Podcast.
Join me as I cover ten PANCE, PANRE, and EOR™ review questions from the Smarty PANCE Instagram/Facebook page and the smartypance.com board review website.
Special from today’s episode:

* Join the Smarty PANCE Member’s Community
* Check out our all-new End of Curriculum™ (EOC) Exam Course (still in development)
* Follow Smarty PANCE and The Daily PANCE Blueprint on Instagram
* Follow Smarty PANCE and The Daily PANCE Blueprint on Facebook

Below you will find an interactive exam to complement today’s podcast.
The Audio PANCE/PANRE and EOR PA Board Review Podcast
I hope you enjoy this free audio component to the examination portion of this site. The full board review course includes over 2,000 interactive board review questions and is available to all members of Smarty PANCE.

* You can download and listen to past FREE episodes here, on iTunes, Spotify, Google Podcasts, Stitcher, and most podcasting apps.
* You can listen to the latest episode, take an interactive quiz, and download more resources below.

Listen Carefully Then Take the Practice Exam
If you can’t see the audio player, click here to listen to the full episode.
Podcast Episode 99: Ten PANCE/PANRE and EOR Topic Blueprint Questions
1. A 42-year-old male on lithium presents with polyuria, nocturia, and polydipsia. Laboratory findings are remarkable for slightly elevated sodium. Which of the following is the most likely diagnosis?
A. Neurogenic diabetes insipidus
B. Nephrogenic diabetes insipidus
C. Type 2 diabetes mellitus
D. SIADH
E. Adrenal insufficiency
2. A 50-year-old female presents with poor appetite, low energy, poor concentration, and feelings of hopelessness on most days for the past 3 years. She denies suicidal ideation. She has never had a past manic or hypomanic episode. Which of the following is the best treatment option?
A. Haloperidol
B. Fluoxetine
C. Lorazepam
D. Amitriptyline
E. Risperidone
3. An 85-year-old male with a history of chronic kidney disease presents to the ER with muscle cramps. Laboratory studies reveal potassium of 7.8 mEq/L. EKG reveals peaked T waves. Which of the following is the best initial med to give?
A. Insulin
B. Albuterol
C. Furosemide
D. Sodium bicarbonate
E. Calcium gluconate
4. A 28-year-old male with sickle cell disease presents to the ER with chest pain, dyspnea, and a cough for the past day. Vitals are remarkable for SpO2 91% and T 102.2F.]]>
The Physician Assistant Life | Smarty PANCE full 25:48 505
Podcast Episode 98: Ten PANCE, PANRE, and Rotation Review Questions http://podcast.thepalife.com/podcast-episode-98/ Tue, 24 May 2022 15:07:00 +0000 http://podcast.thepalife.com/?p=495 Welcome to episode 98 of the Audio PANCE and PANRE Physician Assistant/Associate Board Review Podcast. Join me as I cover ten PANCE, PANRE, and EOR™ review questions from the Smarty PANCE Instagram/Facebook page and the smartypance.com board review website. Special from today’s episode: Join the Smarty PANCE Member’s Community Check out our all-new End of […]

The post Podcast Episode 98: Ten PANCE, PANRE, and Rotation Review Questions appeared first on The Audio PANCE and PANRE.

]]>
The Audio PANCE and PANRE Episode 98 Ten PANCE, PANRE, and Rotation Review Questions

Welcome to episode 98 of the Audio PANCE and PANRE Physician Assistant/Associate Board Review Podcast.

Join me as I cover ten PANCE, PANRE, and EOR™ review questions from the Smarty PANCE Instagram/Facebook page and the smartypance.com board review website.

Special from today’s episode:

Below you will find an interactive exam to complement today’s podcast.

The Audio PANCE/PANRE and EOR PA Board Review Podcast

I hope you enjoy this free audio component to the examination portion of this site. The full board review course includes over 2,000 interactive board review questions and is available to all members of Smarty PANCE.

Listen Carefully Then Take the Practice Exam

If you can’t see the audio player, click here to listen to the full episode.

Podcast Episode 98: Ten PANCE/PANRE and EOR Topic Blueprint Questions

1. An 81-year-old female presents to the ER with acute onset of low back pain. She complains of bowel dysfunction and loss of sensation over her inner thighs. Physical exam reveals decreased lower extremity reflexes. Which of the following is the best diagnostic test for the likely diagnosis?

A. Lumbar radiographs
B. MRI of the lumbosacral spine
C. CSF fluid analysis
D. Scoliosis studies
E. None of the above

Click here to see the answer

The answer is B. MRI of the lumbosacral spine

Cauda equina syndrome is a surgical emergency caused by severe stenosis in the lumbar spine (often due to acute disc herniation).

Clinical manifestations include bowel/bladder dysfunction, decreased lower extremity reflexes, sciatica, saddle anesthesia (loss of sensation over the perineum, buttock, medial aspect of thighs), and decreased anal sphincter tone. Symptoms can develop acutely or chronically.

An emergent MRI of the lumbosacral spine is the preferred diagnostic test. Surgery is the preferred treatment.

Click here to view the Smarty PANCE lesson

Smarty PANCE Content Blueprint Review:

Covered under ⇒ PANCE Blueprint MusculoskeletalSpinal DisordersCauda equina syndrome

Also covered as part of the Emergency Medicine PAEA EOR topic list

2. Which of the following is the most common cause of Cushing syndrome?

A. Iatrogenic
B. Bronchogenic carcinoma
C. Pituitary adenoma
D. Adrenal adenoma
E. Obesity

Click here to see the answer

The answer is A. Iatrogenic

Cushing syndrome is a condition defined by too much cortisol. The most common cause of Cushing syndrome is exogenous steroid therapy (i.e., medical providers prescribing steroids). Other causes include a pituitary ACTH-secreting adenoma, adrenal tumors, and ectopic ACTH production from neoplasms. Classic clinical features include buffalo hump, moon-facies, purple striae, easy bruising, weight gain, depression, and weakness.

Laboratory tests will be remarkable for elevated cortisol. Late night-salivary cortisol or 24-hour urinary free cortisol is typically the initial screening test. A low-dose dexamethasone test can be done. Referral to an endocrinologist is usually done at this point (or even prior).

Click here to view the Smarty PANCE lesson

Smarty PANCE Content Blueprint Review:

Covered under ⇒ PANCE Blueprint EndocrinologyAdrenal DisordersCushing’s syndrome

Also covered as part of the Internal Medicine EOR, Family Medicine EOR, and Emergency Medicine PAEA EOR topic list

3. A 73-year-old female presents with facial flushing, wheezing, and watery diarrhea for a few months. Her symptoms are worsened by certain foods. Which of the following is the best initial diagnostic study to order for the suspected diagnosis?

A. 5-HIAA in urine
B. Serum ACE levels
C. Chest radiograph
D. Upper endoscopy
E. Bronchoscopy

Click here to see the answer

The answer is A. 5-HIAA in urine

Carcinoid tumors are uncommon tumors that originate from neuroendocrine cells and secrete serotonin. The most common site is the appendix, but they can also be found in the lungs, kidney, etc.

Carcinoid syndromes develop 10% of the time; clinical features include flushing, sweating, wheezing, and watery diarrhea. These symptoms are due to the release of serotonin and can be precipitated by foods high in tyramine or ethanol.

The best initial diagnostic study is a 24-hour measurement of urinary excretion of 5-HIAA (Note: 5-HIAA is a degradation product from 5HT). Surgical resection is the definitive treatment of choice.

Click here to view the Smarty PANCE lesson

Smarty PANCE Content Blueprint Review:

Covered under ⇒ PANCE Blueprint PulmonaryPulmonary NeoplasmsCarcinoid tumors

Also covered as part of the Internal Medicine PAEA EOR topic list

4. Which of the following is the most common cause of Mallory-Weiss Syndrome?

A. Iatrogenic
B. Vomiting
C. Abdominal trauma
D. Hiatal hernia
E. Corrosive ingestion

Click here to see the answer

The answer is B. Vomiting

Mallory-Weiss Syndrome (MWS) is defined by a mucosal tear at or below the gastroesophageal junction. MWS is one of the most common causes of an upper GI bleed. It is most commonly due to forceful vomiting. It is classically associated with alcoholism (binge drinking), but really anything that can cause vomiting can lead to a tear. Other causes include iatrogenic, trauma, hiatal hernia, and corrosive ingestion. Diagnosis is made via upper endoscopy. Intervention is rarely necessary, but if needed, the patient can undergo surgery or embolization. Do not confuse this with Boerhaave syndrome, which is an actual perforation of the esophagus!

Click here to view the Smarty PANCE lesson

Smarty PANCE Content Blueprint Review:

Covered under ⇒ PANCE Blueprint GI and NutritionEsophageal DisordersMallory Weiss tear

Also covered as part of the Internal Medicine EOR and Emergency Medicine PAEA EOR topic list

5. A 51-year-old male presents to the clinic complaining of “band-like pressure” around his head. He has been more stressed lately. Which of the following is the most likely diagnosis?

A. Cluster headache
B. Migraine with aura
C. Sinus headache
D. Tension headache
E. Medication overuse headache

Click here to see the answer

The answer is D. Tension headache

A tension-type headache (TTH) is the most common headache. It usually presents with steady, aching, “band-like” pain that circles the entire head. There may also be tightness in the posterior neck muscles. Precipitants include anxiety, depression, and stress.

You should treat any underlying depression and/or anxiety. NSAIDs and acetaminophen are the first-line abortive treatment options for mild-moderate TTHs. For long-term prophylactic treatment, amitriptyline is preferred.

Click here to view the Smarty PANCE lesson

Smarty PANCE Content Blueprint Review:

Covered under ⇒ PANCE Blueprint NeurologyHeadachesTension headache

Also covered as part of the Internal Medicine PAEA EOR topic list

6. Which of the following physical exam findings would you expect in a patient with mitral regurgitation?

A. Holosystolic blowing murmur best heard at the apex with radiation to the axilla
B. Diastolic blowing murmur at the left upper sternal border
C. Systolic ejection crescendo-decrescendo murmur at upper right sternal border
D. Mid-systolic harsh murmur best heard at the left upper sternal border
E. None of the above

Click here to see the answer

The answer is A. Holosystolic blowing murmur best heard at the apex with radiation to the axilla

Mitral regurgitation (MR) is characterized by a blowing, holosystolic murmur at the apex with radiation to the axilla. It can be acute or chronic. Acute causes include endocarditis and papillary muscle rupture; chronic causes include mitral valve prolapse (MVP), rheumatic fever, cardiomyopathy. The most common cause of MR is MVP.

Remember, regurgitation murmurs will often have blowing sounds, while stenotic murmurs are harsh and rumbling. Diagnosis should be made with an echocardiogram. Treatment (repair vs. replace vs. medical therapy) depends on many factors.

Click here to view the Smarty PANCE lesson

Smarty PANCE Content Blueprint Review:

Covered under ⇒ PANCE Blueprint CardiologyValvular DisordersMitral regurgitation

Also covered as part of the Emergency Medicine EOR topic list

7. Which of the following is not an expected clinical manifestation of polyarteritis nodosa?

A. Kidney failure
B. Livedo reticularis
C. pANCA positive
D. Pulmonary fibrosis
E. Hypertension

Click here to see the answer

The answer is D. Pulmonary fibrosis

Polyarteritis nodosa is a systemic vasculitis of medium-sized vessels (specifically involving the nervous system and GI tract) that leads to thrombi and microaneurysms. It is associated with hepatitis B, HIV, and drug reactions. Clinical features include fever, hypertension, livedo reticularis, abdominal pain, and arthralgias. Remember patients may be positive for pANCA, but are ANCA negative.

A definitive diagnosis is made via biopsy of involved tissue (or mesenteric angiography). What sets polyarteritis nodosa apart from other vasculitides is that it spares the lungs. Treatment includes high-dose steroids. The prognosis is not really good.

Click here to view the Smarty PANCE lesson

Smarty PANCE Content Blueprint Review:

Covered under ⇒ PANCE Blueprint MusculoskeletalRheumatologic DisordersPolymyalgia rheumatica

Also covered as part of the Internal Medicine PAEA EOR topic list

8. An 8-year-old female presents to the clinic with her dad. She is complaining of left ear pain and pruritus for the past few days. The physical exam is remarkable for drainage and pain on tragal pressure. The tympanic membrane can’t be visualized. Which of the following is the best treatment option?

A. Oral amoxicillin
B. Neomycin/polymyxin B/hydrocortisone drops
C. Oral cefepime
D. Oral steroids
E. Ciprofloxacin/dexamethasone drops

Click here to see the answer

The answer is E. Ciprofloxacin/dexamethasone drops

The patient has otitis externa (aka swimmer’s ear), which is inflammation of the external auditory canal. The most common bacterial cause is Pseudomonas aeruginosa (~40%). Risk factors include water exposure, trauma, ear devices, and dermatologic conditions. Clinical features include ear pain, pruritus, auricular discharge, pain/tenderness when pressure is applied to the tragus, and hearing loss.

Treatment is topical ear antibiotic/steroid drops. Medication options include neomycin/polymyxin B /hydrocortisone and ciprofloxacin/dexamethasone. **Remember that aminoglycosides are ototoxic and should be avoided when the tympanic membrane can’t be visualized!**

Click here to view the Smarty PANCE lesson

Smarty PANCE Content Blueprint Review:

Covered under ⇒ PANCE Blueprint EENTEar DisordersExternal earOtitis externa

Also covered as part of the Pediatric EOR, Family Medicine EOR, and Emergency Medicine PAEA EOR topic list

9. Which of the following correctly depicts the typical clinical features of Parkinson’s disease?

A. Pill-rolling tremor, bradykinesia, rigidity, masked facies
B. Loss of declarative episodic memory, apraxia, olfactory dysfunction
C. Visual hallucinations, REM sleep behavior disorder, gait issues
D. Hyperorality, apathy, socially inappropriate behavior
E. None of the above

Click here to see the answer

The answer is A. Pill-rolling tremor, bradykinesia, rigidity, masked facies

Parkinson’s disease is a neurodegenerative disease. Its 3 cardinal features are tremor (“pill-rolling” at rest), bradykinesia, and rigidity. Tremor is the presenting feature in most patients. Other clinical features include hypomimia (masked facies), speech impairment, mood disorders (depression or anxiety), dysphagia, shuffling gait, stooped posture, sleep issues, cognitive dysfunction, and autonomic dysfunction.

It is mainly a clinical diagnosis. When patients respond to dopaminergic drugs, this supports the diagnosis. The mainstay of treatment is levodopa or dopamine agonists (pramipexole, ropinirole). For mild disease, monoamine oxidase type B inhibitors or amantadine can be given.

Click here to view the Smarty PANCE lesson

Smarty PANCE Content Blueprint Review:

Covered under ⇒ PANCE Blueprint NeurologyMovement DisordersParkinson’s disease

Also covered as part of the Internal Medicine EOR and Family Medicine PAEA EOR topic list

10. A 72-year-old male presents with an abnormal change in bowel habits and fatigue. The physical exam is remarkable for pale conjunctiva and a palpable abdominal mass. His hemoglobin is 10 mg/dL. Which of the following is likely to be elevated?

A. Alpha-fetoprotein
B. CEA
C. CA-125
D. CA 19-9
E. AAT1

Click here to see the answer

The answer is B. CEA

The patient has colorectal cancer (CRC), which is the third most common cause of cancer death in the U.S. The USPSTF suggests screening at age 45 for individuals with an average risk. Patients may present with alarming “red flag” symptoms, a bowel obstruction, or CRC may just be found via routine screening. The most common symptom is a change in bowel habits (~70%). Other symptoms include rectal bleeding, iron deficiency anemia, and pain.

Initial diagnostic tests include colonoscopy, CBC, fecal occult blood testing, and tumor markers like carcinoembryonic antigen (CEA). CEA is more often used for surveillance and not screening though (since its sensitivity for CRC is only 46%).

Click here to view the Smarty PANCE lesson

Smarty PANCE Content Blueprint Review:

Covered under ⇒ PANCE Blueprint GI and NutritionGastrointestinal System NeoplasmsColon cancer

Also covered as part of the Internal Medicine EOR, Family Medicine EOR, General Surgery PAEA EOR topic list

Looking for all the podcast episodes?

This FREE podcast series is limited to every other episode, you can download and enjoy the complete audio series by becoming a Smarty PANCE member.

I will be releasing new episodes every few weeks. Smarty PANCE is now discounted, so sign up now before it’s too late!

Additional resources and links from the show

This Podcast is available on iOS and Android

Download the Interactive Content Blueprint Checklist

Interactive Content Blueprint for the 2022-2023 PANCE

Follow this link to download your FREE copy of the PANCE/PANRE/EOR™ Content Blueprint Checklists

Print it up and start crossing out the topics you understand, marking the ones you don’t, and making notes of key terms you should remember. The PDF version is interactive and linked directly to the individual lessons on Smarty PANCE.

Smarty PANCE is not sponsored or endorsed by, or affiliated with, the NCCPA or the Physician Assistant Education Association (PAEA). All trademarks are the property of their respective owners.

The post Podcast Episode 98: Ten PANCE, PANRE, and Rotation Review Questions appeared first on The Audio PANCE and PANRE.

]]>
Welcome to episode 98 of the Audio PANCE and PANRE Physician Assistant/Associate Board Review Podcast. Join me as I cover ten PANCE, PANRE, and EOR™ review questions from the Smarty PANCE Instagram/Facebook page and the smartypance.



Welcome to episode 98 of the Audio PANCE and PANRE Physician Assistant/Associate Board Review Podcast.



Join me as I cover ten PANCE, PANRE, and EOR™ review questions from the Smarty PANCE Instagram/Facebook page and the smartypance.com board review website.



Special from today’s episode:



* Join the Smarty PANCE Member’s Community* Check out our all-new End of Curriculum™ (EOC) Exam Course (still in development)* Follow Smarty PANCE and The Daily PANCE Blueprint on Instagram* Follow Smarty PANCE and The Daily PANCE Blueprint on Facebook



Below you will find an interactive exam to complement today’s podcast.



The Audio PANCE/PANRE and EOR PA Board Review Podcast



I hope you enjoy this free audio component to the examination portion of this site. The full board review course includes over 2,000 interactive board review questions and is available to all members of Smarty PANCE.



* You can download and listen to past FREE episodes here, on iTunes, Spotify, Google Podcasts, Stitcher, and most podcasting apps.* You can listen to the latest episode, take an interactive quiz, and download more resources below.



Listen Carefully Then Take the Practice Exam





If you can’t see the audio player, click here to listen to the full episode.



Podcast Episode 98: Ten PANCE/PANRE and EOR Topic Blueprint Questions



1. An 81-year-old female presents to the ER with acute onset of low back pain. She complains of bowel dysfunction and loss of sensation over her inner thighs. Physical exam reveals decreased lower extremity reflexes. Which of the following is the best diagnostic test for the likely diagnosis?



A. Lumbar radiographsB. MRI of the lumbosacral spineC. CSF fluid analysisD. Scoliosis studiesE. None of the above





2. Which of the following is the most common cause of Cushing syndrome?



A. IatrogenicB. Bronchogenic carcinomaC. Pituitary adenomaD. Adrenal adenomaE. Obesity





3. A 73-year-old female presents with facial flushing, wheezing, and watery diarrhea for a few months. Her symptoms are worsened by certain foods. Which of the following is the best initial diagnostic study to order for the suspected diagnosis?



A. 5-HIAA in urineB. Serum ACE levelsC. Chest radiographD. Upper endoscopyE. Bronchoscopy
]]>
The Physician Assistant Life | Smarty PANCE full 17:16 495